Property Case

32
Republic of the Philippines SUPREME COURT SECOND DIVISION G.R. No. 160384. April 29, 2005 CESAR T. HILARIO, for himself and as Attorney-in-Fact of IBARRA, NESTOR, LINA and PRESCILLA, all surnamed HILARIO, Petitioners, vs. ALLAN T. SALVADOR, Respondents. HEIRS OF SALUSTIANO SALVADOR, namely, REGIDOR M. SALVADOR and VIRGINIA SALVADOR-LIM, respondents-intervenors. D E C I S I O N CALLEJO, SR., J.: This is a petition for review on certiorari under Rule 45 of the Revised Rules of Court of the Decision 1 of the Court of Appeals (CA) in CA-G.R. CV No. 63737 as well as its Resolution 2 denying the motion for the reconsideration of the said decision. The Antecedents On September 3, 1996, petitioners Cesar, Ibarra, Nestor, Lina and Prescilla, all surnamed Hilario, filed a complaint with the Regional Trial Court (RTC) of Romblon, Romblon, Branch 71, against private respondent Allan T. Salvador. They alleged therein, inter alia, as follows: 2. That, the plaintiffs are co-owners by inheritance from Concepcion Mazo Salvador of a parcel of land designated as Cad. Lot No. 3113-part, located at Sawang, Romblon, Romblon, which property was [adjudged] as the hereditary share of their father, Brigido M. Hilario, Jr. when their father was still single, and which adjudication was known by the plaintiffs[’] father’s co-heirs; 3. That, sometime in 1989, defendant constructed his dwelling unit of mixed materials on the property of the plaintiffs’ father without the knowledge of the herein plaintiffs or their predecessors-in-interest; 4. That, demands have been made of the defendant to vacate the premises but the latter manifested that he have (sic) asked the prior consent of their grandmother, Concepcion Mazo Salvador; 5. That, to reach a possible amicable settlement, the plaintiffs brought the matter to the Lupon of Barangay Sawang, to no avail, evidenced by the CERTIFICATE TO FILE ACTION hereto attached as ANNEX B; 6. That, the unjustified refusal of the defendant to vacate the property has caused the plaintiffs to suffer shame, humiliation, wounded feelings, anxiety and sleepless nights; 7. That, to protect their rights and interest, plaintiffs were constrained to engage the services of a lawyer. 3 The petitioners prayed that, after due proceedings, judgment be rendered in their favor, thus: WHEREFORE, it is prayed of this Honorable Court that after due process (sic), an order be issued for the defendant to vacate and peacefully turn over to the plaintiffs the occupied property and that defendant be made to pay plaintiffs: a. actual damages, as follows: a.1. transportation expenses in connection with the projected settlement of the case amounting to P 1,500.00 and for the subsequent attendance to the hearing of this case at P 1,500.00 each schedule; a.2. attorney’s fees in the amount of P 20,000.00 and P 500.00 for every court appearance; b. moral and exemplary damages in such amount incumbent upon the Honorable Court to determine; and c. such other relief and remedies just and equitable under the premises. 4 The private respondent filed a motion to dismiss the complaint on the ground of lack of jurisdiction over the nature of the action, citing Section 33 of Batas Pambansa (B.P.) Blg. 129, as amended by Section 3(3) of Republic Act (R.A.) No. 7691. 5 He averred that – (1) the complaint failed to state the assessed value of the land in dispute; (2) the complaint does not sufficiently identify and/or describe the parcel of land referred to as the subject-matter of this action; both of which are essential requisites for determining the jurisdiction of the Court where the case is filed. In this case, however, the assessed value of the land in question is totally absent in the allegations of the complaint and there is nothing in the relief prayed for which can be picked-up for determining the Court’s jurisdiction as provided by law. In the face of this predicament, it can nevertheless be surmised by reading between the lines, that the assessed value of the land in question cannot exceed P 20,000.00 and, as such, it falls within the jurisdiction of the Municipal Trial Court of Romblon and should have been filed before said Court rather than before the RTC. … 6 The petitioners opposed the motion. 7 They contended that the RTC had jurisdiction over the action since the court can take judicial notice of the market value of the property in question, which was P 200.00 per square meter and considering that the property was 14,797 square meters, more or less, the total value thereof is P 3,500,000.00. Besides, according to the petitioners, the motion to dismiss was premature and "the proper time to interpose it is when the [petitioners] introduced evidence that the land is of such value."

Transcript of Property Case

Page 1: Property Case

Republic of the PhilippinesSUPREME COURT

SECOND DIVISION

G.R. No. 160384. April 29, 2005

CESAR T. HILARIO, for himself and as Attorney-in-Fact of IBARRA, NESTOR, LINA and PRESCILLA, all surnamed HILARIO, Petitioners, vs.ALLAN T. SALVADOR, Respondents.

HEIRS OF SALUSTIANO SALVADOR, namely, REGIDOR M. SALVADOR and VIRGINIA SALVADOR-LIM, respondents-intervenors.

D E C I S I O N

CALLEJO, SR., J.:

This is a petition for review on certiorari under Rule 45 of the Revised Rules of Court of the Decision1 of the Court of Appeals (CA) in CA-G.R. CV No. 63737 as well as its Resolution2 denying the motion for the reconsideration of the said decision.

The Antecedents

On September 3, 1996, petitioners Cesar, Ibarra, Nestor, Lina and Prescilla, all surnamed Hilario, filed a complaint with the Regional Trial Court (RTC) of Romblon, Romblon, Branch 71, against private respondent Allan T. Salvador. They alleged therein, inter alia, as follows:

2. That, the plaintiffs are co-owners by inheritance from Concepcion Mazo Salvador of a parcel of land designated as Cad. Lot No. 3113-part, located at Sawang, Romblon, Romblon, which property was [adjudged] as the hereditary share of their father, Brigido M. Hilario, Jr. when their father was still single, and which adjudication was known by the plaintiffs[’] father’s co-heirs;

3. That, sometime in 1989, defendant constructed his dwelling unit of mixed materials on the property of the plaintiffs’ father without the knowledge of the herein plaintiffs or their predecessors-in-interest;

4. That, demands have been made of the defendant to vacate the premises but the latter manifested that he have (sic) asked the prior consent of their grandmother, Concepcion Mazo Salvador;

5. That, to reach a possible amicable settlement, the plaintiffs brought the matter to the Lupon of Barangay Sawang, to no avail, evidenced by the CERTIFICATE TO FILE ACTION hereto attached as ANNEX B;

6. That, the unjustified refusal of the defendant to vacate the property has caused the plaintiffs to suffer shame, humiliation, wounded feelings, anxiety and sleepless nights;

7. That, to protect their rights and interest, plaintiffs were constrained to engage the services of a lawyer.3

The petitioners prayed that, after due proceedings, judgment be rendered in their favor, thus:

WHEREFORE, it is prayed of this Honorable Court that after due process (sic), an order be issued for the defendant to vacate and peacefully turn over to the plaintiffs the occupied property and that defendant be made to pay plaintiffs:

a. actual damages, as follows:

a.1. transportation expenses in connection with the projected settlement of the case amounting to P1,500.00 and for the subsequent attendance to the hearing of this case at P1,500.00 each schedule;

a.2. attorney’s fees in the amount of P20,000.00 and P500.00 for every court appearance;

b. moral and exemplary damages in such amount incumbent upon the Honorable Court to determine; and

c. such other relief and remedies just and equitable under the premises.4

The private respondent filed a motion to dismiss the complaint on the ground of lack of jurisdiction over the nature of the action, citing Section 33 of Batas Pambansa (B.P.) Blg. 129, as amended by Section 3(3) of Republic Act (R.A.) No. 7691.5 He averred that –

(1) the complaint failed to state the assessed value of the land in dispute;

(2) the complaint does not sufficiently identify and/or describe the parcel of land referred to as the subject-matter of this action;

both of which are essential requisites for determining the jurisdiction of the Court where the case is filed. In this case, however, the assessed value of the land in question is totally absent in the allegations of the complaint and there is nothing in the relief prayed for which can be picked-up for determining the Court’s jurisdiction as provided by law.

In the face of this predicament, it can nevertheless be surmised by reading between the lines, that the assessed value of the land in question cannot exceed P20,000.00 and, as such, it falls within the jurisdiction of the Municipal Trial Court of Romblon and should have been filed before said Court rather than before the RTC. …6

The petitioners opposed the motion.7 They contended that the RTC had jurisdiction over the action since the court can take judicial notice of the market value of the property in question, which was P200.00 per square meter and considering that the property was 14,797 square meters, more or less, the total value thereof is P3,500,000.00. Besides, according to the petitioners, the motion to dismiss was premature and "the proper time to interpose it is when the [petitioners] introduced evidence that the land is of such value."

On November 7, 1996, the RTC issued an Order8 denying the motion to dismiss, holding that the action was incapable of pecuniary estimation, and therefore, cognizable by the RTC as provided in Section 19(1) of B.P. Blg. 129, as amended.

After the denial of the motion to dismiss, the private respondent filed his answer with counterclaim.9 Traversing the material allegations of the complaint, he contended that the petitioners had no cause of action against him since the property in dispute was the conjugal property of his grandparents, the spouses Salustiano Salvador and Concepcion Mazo-Salvador.

On April 8, 1997, Regidor and Virginia Salvador filed their Answer-in-Intervention10 making common cause with the private respondent. On her own motion, however, Virginia Salvador was dropped as intervenor.11

During trial, the petitioners adduced in evidence Tax Declaration No. 8590-A showing that in 1991 the property had an assessed value of P5,950.00.12

On June 3, 1999, the trial court rendered judgment finding in favor of the petitioners. The dispositive portion of the decision reads:

WHEREFORE, as prayed for, judgment is rendered:

Ordering the defendant to vacate and peacefully turn over to the plaintiffs the occupied property; and

Page 2: Property Case

Dismissing defendant’s counterclaim.

SO ORDERED.13

Aggrieved, the private respondent and respondent-intervenor Regidor Salvador appealed the decision to the CA, which rendered judgment on May 23, 2003 reversing the ruling of the RTC and dismissing the complaint for want of jurisdiction. The fallo of the decision is as follows:

IN VIEW OF THE FOREGOING, the appealed decision is REVERSED, and the case DISMISSED, without prejudice to its refilling in the proper court.

SO ORDERED.14

The CA declared that the action of the petitioners was one for the recovery of ownership and possession of real property. Absent any allegation in the complaint of the assessed value of the property, the Municipal Trial Court (MTC) had exclusive jurisdiction over the action, conformably to Section 3315 of R.A. No. 7691.

The petitioners filed a motion for reconsideration of the said decision, which the appellate court denied.16 Hence, they filed the instant petition, with the following assignment of errors:

I

THE HONORABLE COURT OF APPEALS COMMITTED GRAVE REVERSIBLE ERROR IN HOLDING THAT THE INSTANT CASE, ACCION REINVINDICATORIA, FALLS WITHIN THE EXCLUSIVE ORIGINAL JURISDICTION OF THE MUNICIPAL TRIAL COURT OF ROMBLON, AND NOT WITH THE REGIONAL TRIAL COURT OF ROMBLON.

II

THE HONORABLE COURT OF APPEALS COMMITTED SERIOUS REVERSIBLE ERROR IN ORDERING THE REFILING OF THE CASE IN THE [PROPER] COURT, INSTEAD OF DECIDING THE CASE ON THE MERITS BASED ON THE COMPLETE RECORDS ELEVATED BEFORE SAID APPELLATE COURT AND IN NOT AFFIRMING IN TOTO THE DECISION OF THE TRIAL COURT.17

The Ruling of the Court

The lone issue for our resolution is whether the RTC had jurisdiction over the action of the petitioners, the plaintiffs in the RTC, against the private respondent, who was the defendant therein.

The petitioners maintain that the RTC has jurisdiction since their action is an accion reinvindicatoria, an action incapable of pecuniary estimation; thus, regardless of the assessed value of the subject property, exclusive jurisdiction falls within the said court. Besides, according to the petitioners, in their opposition to respondent’s motion to dismiss, they made mention of the increase in the assessed value of the land in question in the amount of P3.5 million. Moreover, the petitioners maintain that their action is also one for damages exceeding P20,000.00, over which the RTC has exclusive jurisdiction under R.A. No. 7691.

The petition has no merit.

It bears stressing that the nature of the action and which court has original and exclusive jurisdiction over the same is determined by the material allegations of the complaint, the type of relief prayed for by the plaintiff and the law in effect when the action is filed, irrespective of whether the plaintiffs are entitled to some or all of the claims asserted therein.18 The caption of the complaint is not determinative of the nature of the action. Nor does the jurisdiction of the court depend upon the answer of the defendant or agreement of the parties or to the waiver or acquiescence of the parties.

We do not agree with the contention of the petitioners and the ruling of the CA that the action of the petitioners in the RTC was an accion reinvindicatoria. We find and so rule that the action of the petitioners was an accion publiciana, or one for the recovery of possession of the real property subject matter thereof. An accion reinvindicatoria is a suit which has for its object the recovery of possession over the real property as owner. It involves recovery of ownership and possession based on the said ownership. On the other hand, an accion publiciana is one for the recovery of possession of the right to possess. It is also referred to as an ejectment suit filed after the expiration of one year after the occurrence of the cause of action or from the unlawful withholding of possession of the realty.19

The action of the petitioners filed on September 3, 1996 does not involve a claim of ownership over the property. They allege that they are co-owners thereof, and as such, entitled to its possession, and that the private respondent, who was the defendant, constructed his house thereon in 1989 without their knowledge and refused to vacate the property despite demands for him to do so. They prayed that the private respondent vacate the property and restore possession thereof to them.

When the petitioners filed their complaint on September 3, 1996, R.A. No. 7691 was already in effect. Section 33(3) of the law provides:

Sec. 33. Jurisdiction of Metropolitan Trial Courts, Municipal Trial Courts and Municipal Circuit Trial Courts in Civil Cases. – Metropolitan Trial Courts, Municipal Trial Courts and Municipal Circuit Trial Courts shall exercise:

(3) Exclusive original jurisdiction in all civil actions which involve title to, or possession of, real property, or any interest therein where the assessed value of the property or interest therein does not exceed Twenty Thousand Pesos (P20,000.00) or, in civil actions in Metro Manila, where such assessed value does not exceed Fifty Thousand Pesos (P50,000.00) exclusive of interest, damages of whatever kind, attorney’s fees, litigation expenses and costs: Provided, That in cases of land not declared for taxation purposes, the value of such property shall be determined by the assessed value of the adjacent lots.

Section 19(2) of the law, likewise, provides that:

Sec. 19. Jurisdiction in civil cases. – The Regional Trial Court shall exercise exclusive original jurisdiction:

(2) In all civil actions, which involve the title to, or possession of, real property, or any interest therein, where the assessed value of the property involved exceeds Twenty Thousand Pesos (P20,000.00) or, for civil actions in Metro Manila, where such value exceeds Fifty Thousand Pesos (P50,000.00) except actions for forcible entry into and unlawful detainer of lands or buildings, original jurisdiction over which is conferred upon the Metropolitan Trial Courts, Municipal Trial Courts, and Municipal Circuit Trial Courts.

The jurisdiction of the court over an action involving title to or possession of land is now determined by the assessed value of the said property and not the market value thereof. The assessed value of real property is the fair market value of the real property multiplied by the assessment level. It is synonymous to taxable value.20 The fair market value is the price at which a property may be sold by a seller, who is not compelled to sell, and bought by a buyer, who is not compelled to buy.

Even a cursory reading of the complaint will show that it does not contain an allegation stating the assessed value of the property subject of the complaint.21 The court cannot take judicial notice of the assessed or market value of lands.22 Absent any allegation in the complaint of the assessed value of the property, it cannot thus

Page 3: Property Case

be determined whether the RTC or the MTC had original and exclusive jurisdiction over the petitioners’ action.

We note that during the trial, the petitioners adduced in evidence Tax Declaration No. 8590-A, showing that the assessed value of the property in 1991 was P5,950.00. The petitioners, however, did not bother to adduce in evidence the tax declaration containing the assessed value of the property when they filed their complaint in 1996. Even assuming that the assessed value of the property in 1991 was the same in 1995 or 1996, the MTC, and not the RTC had jurisdiction over the action of the petitioners since the case involved title to or possession of real property with an assessed value of less than P20,000.00.23

We quote with approval, in this connection, the CA’s disquisition:

The determining jurisdictional element for the accion reinvindicatoria is, as RA 7691 discloses, the assessed value of the property in question. For properties in the provinces, the RTC has jurisdiction if the assessed value exceeds P20,000, and the MTC, if the value is P20,000 or below. An assessed value can have reference only to the tax rolls in the municipality where the property is located, and is contained in the tax declaration. In the case at bench, the most recent tax declaration secured and presented by the plaintiffs-appellees is Exhibit B. The loose remark made by them that the property was worth 3.5 million pesos, not to mention that there is absolutely no evidence for this, is irrelevant in the light of the fact that there is an assessed value. It is the amount in the tax declaration that should be consulted and no other kind of value, and as appearing in Exhibit B, this is P5,950. The case, therefore, falls within the exclusive original jurisdiction of the Municipal Trial Court of Romblon which has jurisdiction over the territory where the property is located, and not the court a quo.24

It is elementary that the tax declaration indicating the assessed value of the property enjoys the presumption of regularity as it has been issued by the proper government agency.25

Unavailing also is the petitioners’ argumentation that since the complaint, likewise, seeks the recovery of damages exceeding P20,000.00, then the RTC had original jurisdiction over their actions. Section 33(3) of B.P. Blg. 129, as amended, quoted earlier, explicitly excludes from the determination of the jurisdictional amount the demand for "interest, damages of whatever kind, attorney’s fees, litigation expenses, and costs." This Court issued Administrative Circular No. 09-94 setting the guidelines in the implementation of R.A. No. 7691, and paragraph 2 thereof states that –

2. The exclusion of the term "damages of whatever kind" in determining the jurisdictional amount under Section 19(8) and Section 33(1) of B.P. Blg. 129, as amended by R.A. 7691, applies to cases where the damages are merely incidental to or a consequence of the main cause of action. However, in cases where the claim for damages is the main cause of action, or one of the causes of action, the amount of such claim shall be considered in determining the jurisdiction of the court.

Neither may the petitioners find comfort and solace in Section 19(8) of B.P. Blg. 129, as amended, which states:

SEC. 19. Jurisdiction in civil cases. – Regional Trial Courts shall exercise exclusive original jurisdiction:

(8) In all other cases in which the demand, exclusive of interest, damages of whatever kind, attorney's fees, litigation expenses, and costs or the value of the property in controversy exceeds One Hundred Thousand Pesos (P100,000.00) or, in such other cases in Metro Manila, where the demand, exclusive of the above-mentioned items exceeds Two Hundred Thousand Pesos (P200,000.00).

The said provision is applicable only to "all other cases" other than an action involving title to, or possession of real property in which the assessed value is the controlling factor in determining the court’s jurisdiction. The said damages are merely incidental to, or a consequence of, the main cause of action for recovery of possession of real property.26

Since the RTC had no jurisdiction over the action of the petitioners, all the proceedings therein, including the decision of the RTC, are null and void. The complaint should perforce be dismissed.27

WHEREFORE, the petition is DENIED. The assailed Decision and Resolution of the Court of Appeals in CA-G.R. CV No. 63737 are AFFIRMED. Costs against the petitioners.

SO ORDERED.

Page 4: Property Case

Republic of the PhilippinesSUPREME COURT

Manila

THIRD DIVISION

G.R. No. 156360            January 14, 2005

CESAR SAMPAYAN, petitioner, vs.The HONORABLE COURT OF APPEALS, CRISPULO VASQUEZ and FLORENCIA VASQUEZ GILSANO, respondents.

D E C I S I O N

GARCIA, J.:

In this verified petition for review on certiorari under Rule 45 of the Rules of Court, petitioner Cesar Sampayan seeks the annulment and setting aside of the following issuances of the Court of Appeals in CA-G.R. SP No. 43557, to wit:

1. Decision dated May 16, 2002, denying his petition for review and affirming an earlier decision of the Regional Trial Court at Agusan del Sur, Branch VII, which in turn reversed on appeal a favorable judgment of the Municipal Circuit Trial Court (MCTC) of Bayugan and Sibagat, Agusan del Sur in a forcible entry case thereat commenced against him by herein private respondents, the brother-and-sister Crispulo Vasquez and Florencia Vasquez-Gilsano; and

2. Resolution dated November 7, 2002, which denied his motion for reconsideration.

From the pleadings and memoranda respectively filed by the parties, the Court gathers the following factual antecedents:

On July 8, 1992, in the MCTC of Bayugan and Sibagat, Agusan del Sur, the siblings Crispulo Vasquez and Florencia Vasquez-Gilsano filed complaint for forcible entry against Cesar Sampayan for allegedly having entered and occupied a parcel of land, identified as Lot No. 1959, PLS-225, and built a house thereon without their knowledge, consent or authority, the entry having been supposedly effected through strategy and stealth.

In their complaint, the plaintiffs (now private respondents), substantially alleged that their mother Cristita Quita was the owner and actual possessor of Lot No. 1959; that after their mother’s death on January 11, 1984, they became co-owners pro-indiviso and lawful possessors of the same lot; that on June 1, 1992, while they were temporarily absent from the lot in question, defendant Cesar Sampayan, through strategy and stealth, entered the lot and built a house thereon, to their exclusion; and that, despite their repeated demands for Sampayan to vacate the lot and surrender the possession thereof to them, the latter failed and refused to do so.

In his answer, defendant Sampayan denied the material allegations of the complaint and averred that neither the plaintiffs nor their mother have ever been in possession of Lot No. 1959 and that he does not even know plaintiffs’ identities or their places of residence. He claimed that he did not enter the subject lot by stealth or strategy because he asked and was given permission therefor by Maria Ybañez, the overseer of the lot’s true owners, Mr. and Mrs. Anastacio Terrado who were then temporarily residing in Cebu City for business purposes. In the same answer, Sampayan alleged that the plaintiffs’ claim has long prescribed for the reason that the lot in dispute had been possessed and declared for taxation purposes by the spouses Felicisimo Oriol and Concordia Balida-Oriol in 1960, and that in 1978, the Oriol spouses sold one-half (1/2) of the lot to the spouses Mr. and Mrs. Anastacio Terrado, while the other half, to the couple Manolito Occida and Juliana Sambale-Occida in 1979. Both vendees, so Sampayan averred, have actually possessed the respective portions

purchased by them up to the present. He thus prayed for the dismissal of the complaint.

In the ensuing proceedings following the joinder of issues, the plaintiffs, to prove that they have been in actual possession of Lot No. 1959 when defendant Sampayan effected his entry thereto, submitted in evidence the following documents:

1. Tax Declaration No. 3180 in the name of Cristita Quita;

2. Certificate of Death showing the date of death of Cristita Quita on January 11, 1984;

3. Certificate issued by Fermina R. Labonete, Land Management Officer-III of CENRO X-3-E, DENR-X-3-9, Bayugan, Agusan del Sur showing that Lot 1959, PLS-225 is covered by a Miscellaneous Sales Application of Cristita Quita;

4. Affidavit of one Emiliano G. Gatillo to the effect that he was the one who gave the lot in question to Cristita Quita sometime in 1957 and that since then the latter had been occupying the lot;

Plaintiffs also filed a Supplemental Position Paper dated July 13, 1994 for the purpose of showing that Cristita Quita is one of the oppositors in Cadastral Case No. 149. Together with said position paper, they submitted a copy of the Answer/Opposition earlier filed in Cadastral Case No. 149. In said cadastral case, Cristita Quita was claiming Lot 1959, thus her name appeared in the list of oppositors therein.

5. The decision in the said Cadastral Case No. 149 showing that the then Court of First Instance of Agusan del Sur declared Lot No. 1959 as one of the lots subject of the same cadastral case.

For his part, defendant Sampayan, to prove the allegations in his answer, offered in evidence the following:

1. Tax Declaration No. A-11698 in the name of Felicisimo Oriol, which cancels Tax Declaration 8103;

2. Tax Declaration No. GRB-01-930 in the name of Felicisimo Oriol which cancels Tax Declaration No. A-11698;

3. Deed of Absolute Sale of Portion of Land, dated April 30, 1979, executed by Jesus Oriol for and in behalf of the spouses Felicisimo Oriol and Concordia Balida-Oriol, conveying the one-half (1/2) portion of Lot No. 1959 to the couple Manolito Occida and Juliana Sambale-Occida who possessed the one-half (1/2) portion and introduced improvements thereon, such as coconut and caimito trees;

4. Deed of Relinquishment of Rights of Portion of Land, executed by the spouses Oriol in favor of the same couple Manolito Occida and Juliana Sambale-Occida, to further strengthen the transfer of possession and whatever possessory rights the Oriols had in the lot in question;

5. Deed of Absolute Sale of Land executed by Concordia Balida-Oriol with the conformity of Teodosio Mosquito (another claimant), to prove that the other half of Lot No. 1959 was sold in 1978 to Mr. and Mrs. Anastacio Terrado whose overseer allowed Sampayan to enter and occupy the premises;

6. Protest filed with the CENRO, Agusan del Sur by the vendee Juliana Sambale-Occida against the Miscellaneous Sales Application of Cristita Quita;

Page 5: Property Case

7. Affidavit of Dionesia Noynay attesting to the fact that she is residing in Lot No. 1957, a lot adjacent to the lot in question, since 1960 up to the present. In the same affidavit, Dionisia claimed that neither Cristita Quita, much less the plaintiffs, had ever possessed Lot No. 1959. She claimed that it was the Occida couple who possessed said lot and introduced improvements thereon; and

8. Affidavit of Juliana Occida and Maria Ybañez to show the impossibility of plaintiffs’ possession of the same lot.

Meanwhile, on March 21, 1996, while the case was pending with the MCTC, the presiding judge thereof personally conducted an ocular inspection of the contested lot in the presence of the parties and/or their counsels. Among those found in the area during the inspection are: the house of defendant Sampayan; the dilapidated house of a certain Peter Siscon; and a portion of the house of Macario Noynay, husband of Dionisia Noynay, one of Sampayan’s witnesses.

Based on his ocular findings, the judge concluded that the improvements he saw in the premises could never have been introduced by the plaintiffs nor by their mother Cristita Quita but by the vendees of the same lot. Reproduced by petitioner Jose Sampayan in the instant petition as well as in the Memorandum he subsequently filed with this Court, the MCTC judge’s findings and observations during the ocular inspection, about which the herein private respondents took no exception whatsoever, are hereunder quoted, as follows:

"Noted inside the land are the house of the defendant, Cesar Sampayan, of Peter Siscon, which appears to be dilapidated, and part of the house of Macario Noynay which encroached to the land in question. Planted on the land are five (5) coconut trees, fruit bearing, three (3) not fruit bearing coconut trees, and three (3) star apple or caimito trees. Defendant Sampayan admitted that he started occupying the land since 1992. It is admitted by the parties during the ocular inspection that one-half (1/2) portion of the land was bought by a certain Occida from certain Mr. and Mrs. Felicisimo Oriol.

The findings in the ocular inspection have confirmed the allegation of the defendant that his predecessors-in-interest have introduced improvements by planting caimito trees, coconut trees, and others on the land in question.

Nothing can be seen on the land that plaintiffs had once upon a time been in possession of the land. The allegation that Cristita Quita, the predecessor-in-interest of the plaintiffs had been in possession of the said property since 1957, openly, exclusively, continuously, adversely and in the concept of an owner is a naked claim, unsupported by any evidence.

Clearly, from the appearance of the improvements introduced by the predecessors-in-interest of the defendant, it is showed that they have been in possession of the land for more than one (1) year. Hence, the action of the plaintiffs, if any, is accion publiciana or plenaria de possession"1 (Emphasis supplied).

In time, the MCTC rendered judgment dismissing the compliant "for lack of merit".

Therefrom, the plaintiffs appealed to the Regional Trial Court (RTC) at Agusan del Sur, which appeal was raffled to Branch VII thereof. In a decision dated December 5, 1996, said court reversed that of the MCTC, taking note of the fact that Cristita Quita was among the oppositors in Cadastral Case No. 149 and that she filed a Miscellaneous Sales Application over the lot. On the basis of such finding, the RTC concluded that it was Cristita Quita, predecessor-in-interest of the herein private respondents, who was in actual prior physical possession of Lot No. 1959.

Unable to accept the RTC judgment, Sampayan went to the Court of Appeals on a petition for review, thereat docketed as CA-G.R. SP No. 43557.

As stated at the threshold hereof, the Court of Appeals, in the herein assailed Decision dated May 16, 2002,2 denied Sampayan’s petition. His motion for reconsideration having been similarly denied by that court in its Resolution of November 7, 2002,3 Sampayan is now with us via the present recourse, it being his submissions -

"I.

THAT THE COURT OF APPEALS ERRED IN RULING THAT THE MUNICIPAL CIRCUIT TRIAL COURT OF BAYUGAN, AGUSAN DEL SUR, HAS JURISDICTION OVER THE CASE, CONSIDERING THAT DURING THE HEARING THEREOF IT WAS FOUND OUT BY THE SAID MUNICIPAL COURT THAT ACCION PUBLICIANA OR PLENARIA DE POSESION, AND NOT FORCIBLE ENTRY, IS THE PROPER ACTION;

II.

THAT THE CONCLUSION OF THE HONORABLE COURT OF APPEALS THAT PRIVATE RESPONDENTS HAVE BEEN IN PRIOR ACTUAL POSSESSION IS CONTRADICTED BY EVIDENCE ON RECORD, AND CONSIDERING THAT THE POSSESSION TO BE LEGALLY SUFFICIENT, CONSIST (SIC) IN THE EXERCISE OF DOMINIUM OVER IT, SUCH AS FENCING, CULTIVATING OR OTHER UNMISTAKABLE ACTS OF EXCLUSIVE CUSTODY AND CONTROL – FACTS WHICH THE PRIVATE RESPONDENTS HAVE NEVER DONE - IS CONTRARY TO LAW".4

In the main, petitioner maintains that based on the pieces of evidence on record, he had sufficiently proven his prior physical possession of the subject lot. Upon this premise, he argues that private respondents’ complaint for forcible entry has no leg to stand on, adding that the proper remedy available to the latter is accion publiciana or plenaria de posesion which falls under the original jurisdiction of Regional Trial Courts and not of Municipal Circuit Trial Courts.

As we see it, the arguments put forward by the petitioner crystallize to one pivotal question: will the complaint for forcible entry in this case prosper? To resolve this, however, we must first determine as to who between the herein parties was in prior actual physical possession of the subject lot at the time the complaint was filed in the MCTC. For, as we have said in Gaza vs. Lim 5 ,

"xxx In an action for forcible entry, the plaintiff must prove that he was in prior possession of the land or building and that he was deprived thereof by means of force, intimidation, threat, strategy or stealth. xxx"

We emphasize, absence of prior physical possession by the plaintiff in a forcible entry case warrants the dismissal of his complaint.1a\^/phi1.net

Undoubtedly, the issue of prior physical possession is one of fact, and settled is the rule that this Court is not a trier of facts and does not normally embark on a re-examination of the evidence adduced by the parties during trial. Of course, the rule admits of exceptions. So it is that in Insular Life Assurance Company, Ltd. vs. CA ,6 we wrote:

"[i]t is a settled rule that in the exercise of the Supreme Court's power of review, the Court is not a trier of facts and does not normally undertake the re-examination of the evidence presented by the contending parties' during the trial of the case considering that the findings of facts of the CA are conclusive and binding on the Court. However, the Court had recognized several exceptions to this rule, to wit: (1) when the findings are grounded entirely on speculation, surmises or conjectures; (2) when the inference made is manifestly mistaken, absurd or impossible; (3) when there is grave abuse of discretion; (4) when the judgment is based on a misapprehension of facts; (5) when the findings of facts are conflicting; (6) when in making its findings the Court of Appeals went beyond the issues of the case, or its findings are contrary to the admissions of both the appellant and the appellee; (7) when the findings are contrary to the trial court; (8) when the findings are

Page 6: Property Case

conclusions without citation of specific evidence on which they are based; (9) when the facts set forth in the petition as well as in the petitioner's main and reply briefs are not disputed by the respondent; (10) when the findings of fact are premised on the supposed absence of evidence and contradicted by the evidence on record; and (11) when the Court of Appeals manifestly overlooked certain relevant facts not disputed by the parties, which, if properly considered, would justify a different conclusion."l^vvphi1.net

To our mind, exceptions (5) and (11) are present in this case.

However, before delving into the question of who as between the petitioner and private respondents had prior physical possession of the subject lot, we deem it best to first resolve the issue of whether or not the MCTC had jurisdiction over the complaint filed in this case, an issue also raised by the petitioner.

Relying on the conclusion of the MCTC that private respondents’ proper remedy is accion publiciana or plenaria de posesion, and not forcible entry, petitioner would deny the MCTC’s jurisdiction over the case.

Petitioner is in error.

In Sarmiento vs. CA7 , we held:

"[t]o give the court jurisdiction to effect the ejectment of an occupant or deforciant on the land, it is necessary that the complaint should embody such a statement of facts as brings the party clearly within the class of cases for which the statutes provide a remedy, as these proceedings are summary in nature. The complaint must show enough on its face to give the court jurisdiction without resort to parol testimony. The jurisdictional facts must appear on the face of the complaint. x x x"

Clear it is from the above that for the MCTC to acquire jurisdiction over a forcible entry case, it is enough that the complaint avers the jurisdictional facts, i.e. that the plaintiff had prior physical possession and that he was deprived thereof by the defendant through force, intimidation, threats, strategy and stealth.8 The complaint in this case makes such an averment. Hence, the irrelevant circumstance that the evidence adduced during the hearing rendered improper an action for forcible entry is of no moment and cannot deprive the MCTC of its jurisdiction over the case. The MCTC continues to have that jurisdiction.

We shall now address the more decisive question of prior physical possession.

After a careful evaluation of the evidence at hand, we find for the petitioner.

To begin with, we are at once confronted by the uncontested findings of the MCTC judge himself during his ocular inspection of the premises in dispute that what he saw thereat "confirmed the allegations of the defendant [now petitioner Sampayan] that his predecessors-in-interest have introduced improvements by planting caimito trees, coconut trees, and others on the land in question", adding that "[N]othing can be seen on the land that plaintiff had once upon a time been in possession of the land", and categorically stating that "[T]he allegation that Cristita Quita, the predecessor-in-interest of the plaintiffs had been in possession of the said property since 1957, openly, exclusively, continuously, adversely and in the concept of an owner is a naked claim, unsupported by any evidence".1awphi1.nét

Then, too, there is the sworn affidavit of Dionesia Noynay to the effect that she had been residing since 1960 onward on Lot No. 1957, the lot adjacent to Lot No. 1959, and that neither the private respondents nor their mother had ever possessed Lot No. 1959. Coming as it does from an immediate neighbor, Dionesia’s statement commands great weight and respect. Incidentally, the MCTC judge himself found during the ocular inspection that a portion of the house of Macario Noynay, husband of Dionesia, protruded on Lot No. 1959.

We note that in the herein assailed decision, the Court of Appeals attached much significance to the fact that private respondents’ mother Cristita Quita was an oppositor in Cadastral Case No. 149. We rule and so hold that the mother’s being an oppositor in said cadastral case does not, by itself, establish prior physical possession because not all oppositors in cadastral cases are actual possessors of the lots or lands subject thereof.

WHEREFORE, the instant petition is hereby GRANTED and the Decision and Resolution, respectively dated May 16, 2002 and November 7, 2002, of the Court of Appeals REVERSED and SET ASIDE.

SO ORDERED.

Panganiban, (Chairman), Sandoval-Gutierrez, and Corona, Carpio-Morales, JJ., concur.

Page 7: Property Case

Republic of the PhilippinesSUPREME COURT

Manila

THIRD DIVISION

G.R. No. 137013             May 6, 2005

RUBEN SANTOS, petitioner, vs.SPOUSES TONY AYON and MERCY AYON, respondents.

D E C I S I O N

SANDOVAL-GUTIERREZ, J.:

For our resolution is the petition for review on certiorari assailing the Decision1 of the Court of Appeals dated October 5, 1998 in CA-G.R. SP No. 4735 and its Resolution2 dated December 11, 1998 denying the motion for reconsideration.

The petition alleges that on November 6, 1996, Ruben Santos, petitioner, filed with the Municipal Trial Court in Cities (MTCC), Branch 2, Davao City a complaint for illegal detainer against spouses Tony and Mercy Ayon, respondents, docketed as Civil Case No. 3506-B-96.

In his complaint, petitioner averred that he is the registered owner of three lots situated at Lanzona Subdivision, Matina, Davao City, covered by Transfer Certificates of Title (TCT) Nos. 108174, 108175, and 108176. Respondent spouses are the registered owners of an adjacent parcel of land covered by TCT No. T-247792. The previous occupant of this property built a building which straddled both the lots of the herein parties. Respondents have been using the building as a warehouse.

Petitioner further alleged in his complaint that in 1985, when he bought the three lots, he informed respondents that the building occupies a portion of his land. However, he allowed them to continue using the building. But in 1996, he needed the entire portion of his lot, hence, he demanded that respondents demolish and remove the part of the building encroaching his property and turn over to him their possession. But they refused. Instead, they continued occupying the contested portion and even made improvements on the building. The dispute was then referred to the barangay lupon, but the parties failed to reach an amicable settlement. Accordingly, on March 27, 1996, a certification to file action was issued.

In their answer, respondents sought a dismissal of this case on the ground that the court has no jurisdiction over it since there is no lessor-lessee relationship between the parties. Respondents denied they were occupying petitioner's property by mere tolerance, claiming they own the contested portion and have been occupying the same long before petitioner acquired his lots in 1985.

On July 31, 1997, the MTCC rendered its Decision in favor of petitioner, thus:

"WHEREFORE, judgment is rendered in favor of the plaintiff and against the defendants ordering the latter, their successors-in-interest and other persons acting in their behalf to vacate the portion of the subject properties and peacefully surrender possession thereof to plaintiff as well as dismantle/remove the structures found thereon.

Defendants are further ordered to pay reasonable value for the use and occupation of the encroached area in the amount of One Thousand Pesos (P1,000.00) a month beginning September 1996 and the subsequent months thereafter until premises are vacated; to pay attorney's fees of Ten Thousand Pesos (P10,000.00); and to pay the costs of suit.

SO ORDERED."3

On appeal, the Regional Trial Court (RTC), Branch 11, Davao City, in its Decision dated February 12, 1998 in Civil Case No. 25, 654-97, affirmed in toto the MTCC judgment.4 The RTC upheld the finding of the MTCC that respondents' occupation of the contested portion was by mere tolerance. Hence, when petitioner needed the same, he has the right to eject them through court action.

Respondents then elevated the case to the Court of Appeals through a petition for review. In its Decision dated October 5, 1988 now being challenged by petitioner, the Court of Appeals held that petitioner's proper remedy should have been an accion publiciana before the RTC, not an action for unlawful detainer, thus:

"In this case, petitioners were already in possession of the premises in question at the time private respondent bought three (3) lots at the Lanzona Subdivision in 1985, a portion of which is occupied by a building being used by the former as a bodega. Apart from private respondent's bare claim, no evidence was alluded to show that petitioners' possession was tolerated by (his) predecessor-in-interest. The fact that respondent might have tolerated petitioners' possession is not decisive. What matters for purposes of determining the proper cause of action is the nature of petitioners' possession from its inception. And in this regard, the Court notes that the complaint itself merely alleges that defendants-petitioners have been 'occupying a portion of the above properties of the plaintiff for the past several years by virtue of the tolerance of the plaintiff.' Nowhere is it alleged that his predecessor likewise tolerated petitioners' possession of the premises. x x x.

Consequently, x x x, respondent should present his claim before the Regional Trial Court in an accion publiciana and not before the Municipal Trial Court in a summary proceeding of unlawful detainer.

WHEREFORE, the decision under review is hereby REVERSED and SET ASIDE. Accordingly, the complaint for unlawful detainer is ordered DISMISSED."5

Petitioner filed a motion for reconsideration, but was denied by the Appellate Court in its Resolution dated December 11, 1998.

Hence, the instant petition for review on certiorari ascribing to the Court of Appeals the following errors:

"I

THE HONORABLE COURT OF APPEALS MISAPPLIED THE LAW IN DISMISSING THE INSTANT CASE ON THE GROUND THAT PETITIONER SHOULD PRESENT HIS CLAIM BEFORE THE REGIONAL TRIAL COURT IN AN ACCION PUBLICIANA.

II

THE FINDINGS OF THE HONORABLE COURT OF APPEALS IS NOT IN CONSONANCE WITH EXISTING LAWS AND JURISPRUDENCE."

The sole issue here is whether the Court of Appeals committed a reversible error of law in holding that petitioner's complaint is within the competence of the RTC, not the MTCC.

Petitioner contends that it is not necessary that he has prior physical possession of the questioned property before he could file an action for unlawful detainer. He stresses that he tolerated respondents' occupancy of the portion in controversy until he needed it. After his demand that they vacate, their continued possession became illegal. Hence, his action for unlawful detainer before the MTCC is proper.

Respondents, in their comment, insisted that they have been in possession of the disputed property even before petitioner purchased the same on April 10, 1985.

Page 8: Property Case

Hence, he cannot claim that they were occupying the property by mere tolerance because they were ahead in time in physical possession.

We sustain the petition.

It is an elementary rule that the jurisdiction of a court over the subject matter is determined by the allegations of the complaint and cannot be made to depend upon the defenses set up in the answer or pleadings filed by the defendant.6 This rule is no different in an action for forcible entry or unlawful detainer.7 All actions for forcible entry or unlawful detainer shall be filed with the proper Metropolitan Trial Courts, the Municipal Trial Courts and the Municipal Circuit Trial Courts, which actions shall include not only the plea for restoration of possession but also all claims for damages and costs arising therefrom.8 The said courts are not divested of jurisdiction over such cases even if the defendants therein raises the question of ownership over the litigated property in his pleadings and the question of possession cannot be resolved without deciding the issue of ownership.9

Section 1, Rule 70 on forcible entry and unlawful detainer of the 1997 Rules of Civil Procedure, as amended, reads:

"Section 1. Who may institute proceedings, and when. – Subject to the provisions of the next succeeding section, a person deprived of the possession of any land or building by force, intimidation, threat, strategy, or stealth, or a lessor, vendor, vendee, or other person against whom the possession of any land or building is unlawfully withheld after the expiration or termination of the right to hold possession, by virtue of any contract, express or implied, or the legal representatives or assigns of any such lessor, vendor, vendee or other person may, at any time within one (1) year after such unlawful deprivation or withholding of possession, bring an action in the proper Municipal Trial Court against the person or persons unlawfully withholding or depriving of possession, or any person or persons claiming under them, for the restitution of such possession, together with damages and costs."

Under the above provision, there are two entirely distinct and different causes of action, to wit: (1) a case for forcible entry, which is an action to recover possession of a property from the defendant whose occupation thereof is illegal from the beginning as he acquired possession by force, intimidation, threat, strategy or stealth; and (2) a case for unlawful detainer, which is an action for recovery of possession from defendant whose possession of the property was inceptively lawful by virtue of a contract (express or implied) with the plaintiff, but became illegal when he continued his possession despite the termination of his right thereunder.10

Petitioner's complaint for unlawful detainer in Civil Case No. 3506-B-96 is properly within the competence of the MTCC. His pertinent allegations in the complaint read:

"4. That defendants (spouses) have constructed an extension of their residential house as well as other structures and have been occupying a portion of the above PROPERTIES of the plaintiff for the past several years by virtue of the tolerance of the plaintiff since at the time he has no need of the property;

5. That plaintiff needed the property in the early part of 1996 and made demands to the defendants to vacate and turn over the premises as well as the removal (of) their structures found inside the PROPERTIES of plaintiff; that without any justifiable reasons, defendants refused to vacate the portion of the PROPERTIES occupied by them to the damage and prejudice of the plaintiff.

6. Hence, plaintiff referred the matter to the Office of the Barangay Captain of Matina Crossing 74-A, Davao City for a possible settlement sometime in the latter part of February 1996. The barangay case reached the Pangkat

but no settlement was had. Thereafter, a 'Certification To File Action' dated March 27, 1996 was issued x x x;

x x x."11 (underscoring ours)

Verily, petitioner's allegations in his complaint clearly make a case for an unlawful detainer. We find no error in the MTCC assuming jurisdiction over petitioner's complaint. A complaint for unlawful detainer is sufficient if it alleges that the withholding of the possession or the refusal to vacate is unlawful without necessarily employing the terminology of the law.12 Here, there is an allegation in petitioner's complaint that respondents occupancy on the portion of his property is by virtue of his tolerance. Petitioner's cause of action for unlawful detainer springs from respondents' failure to vacate the questioned premises upon his demand sometime in 1996. Within one (1) year therefrom, or on November 6, 1996, petitioner filed the instant complaint.

It bears stressing that possession by tolerance is lawful, but such possession becomes unlawful when the possessor by tolerance refuses to vacate upon demand made by the owner. Our ruling in Roxas vs. Court of Appeals13 is applicable in this case: "A person who occupies the land of another at the latter's tolerance or permission, without any contract between them, is necessarily bound by an implied promise that he will vacate upon demand, failing which, a summary action for ejectment is the proper remedy against him."

WHEREFORE, the petition is GRANTED. The assailed Decision and Resolution of the Court of Appeals in CA-G.R. SP No. 47435 are hereby REVERSED and SET ASIDE. The Decision dated February 12, 1998 of the Regional Trial Court, Branch 11, Davao City in Civil Case No. 25, 654-97, affirming the Decision dated July 31, 1997 of the Municipal Trial Court in Cities, Branch 2, Davao City in Civil Case No. 3506-B-96, is hereby REINSTATED.

SO ORDERED.

Panganiban, (Chairman), Corona, Carpio-Morales, and Garcia, JJ., concur.

Page 9: Property Case

Republic of the PhilippinesSUPREME COURT

Manila

FIRST DIVISION

G.R. No. 150755               June 28, 2005

RENE GANILA,* EDUARDO DUMADA-OG, SR., RAFAEL GANILA, JOSE PASTRANA, LOURDES GANILA, FLORENTINO GANILA, SERAFIN GANILA, LORETO ARELLANO, CONRADO GANILA, VIVENCIO ALVIOR, EDUARDO GANTALA, AMPARO VILLANUEVA, ELEUTERIO SILVA, ADELINA GANILA, FELIZARDO GANILA, SR., ENRIQUE GANILA, ABRAHAM TANONG, EMILIO ALFARAS, JR., BAPTIST CHRISTIAN LEARNING CENTER, petitioners, vs.HON. COURT OF APPEALS AND VIOLETA C. HERRERA, respondents.

D E C I S I O N

QUISUMBING, J.:

For review on certiorari are the D E C I S I O N1 dated March 30, 2001 of the Court of Appeals in CA-G.R. SP No. 58191, and its Resolution2 dated October 18, 2001 denying the motion for reconsideration. The assailed decision denied the petition to set aside the Resolution3 of the Regional Trial Court (RTC) of San Miguel, Jordan, Guimaras, Branch 65, affirming the Order of the Municipal Circuit Trial Court (MCTC) for the 19 petitioners to vacate the contested parcel of land.

The facts are as follows:

On March 19, 1997, private respondent Violeta Herrera filed 21 ejectment Complaints4 before the 16th MCTC, Jordan-Buenavista-Nueva Valencia, Jordan, Guimaras. Private respondent alleged that she owns Lot 1227 of the Cadastral Survey of Jordan, Guimaras, with an area of 43,210 square meters; that she inherited the lot from her parents; and that she only tolerated petitioners to construct residential houses or other improvements on certain portions of the lot without rental. Sometime in September or October 1996, private respondent demanded that the petitioners vacate the lot and remove their houses and other improvements thereon. Petitioners refused, despite offer of money by way of assistance to them. After the barangay conciliation failed, private respondent filed the complaints.

In their Answers,5 eight6 of the petitioners claimed that Lot 1227 was formerly a shoreline which they developed when they constructed their respective houses. Another eight7 maintained that their houses stood on Lot 1229 of the Cadastral Survey of Jordan, Guimaras. The other three8 asserted that Lot 1227 is a social forest area.

At the preliminary conference, the parties agreed to designate two geodetic engineers as commissioners of the MCTC to conduct a relocation survey of Lot 1227 and to identify who among the petitioners have houses within the lot.9

The commissioners reported that: (1) the house of Henry Gabasa, defendant in Civil Case No. 288-J, is almost outside Lot 1227; (2) the house of Ludovico Amatorio, defendant in Civil Case No. 289-J, diagonally traversed the boundary; and (3) the houses of the 19 petitioners are inside Lot 1227.10

Eight months after herein petitioners’ failure to comment on the manifestation of private respondent to terminate the preliminary conference, the MCTC terminated the preliminary conference.11 Thereafter, petitioners’ counsel Atty. Nelia Jesusa L. Gonzales failed to file her clients’ position papers and affidavits, even after they sought a 30-day extension to file the same.12

Consequently, the MCTC decided the cases as follows:

WHEREFORE, premises considered, judgment is hereby rendered in favor of the plaintiff whereby each of the twenty-one (21) defendants are hereby ordered:

1. To vacate Lot 1227 of the Cadastral Survey of Jordan, Guimaras;

2. To pay Two Hundred Pesos (P200.00) per month from October, 1996 as compensation for the use of the property until the same is vacated; and

3. To pay Two Thousand Pesos (P2,000.00) as attorney’s fees and litigation expenses.

SO ORDERED.13

Petitioners appealed to the RTC, Branch 65, at Jordan, Guimaras, which decided as follows:

WHEREFORE, premises considered, the decision in Civil Cases Nos. 0270-J, 0272-J, 0273-J, 0274-J, 0275-J, 0276-J, 0277-J, 0278-J, 0279-J, 0280-J, 0281-J, 0282-J, 0283-J, 0284-J, 0285-J, 0286-J, 0287-J, 0291-J and 0292-J are hereby affirmed.

The decision of the court below in Civil Cases Nos. 0288-J and 0289-J are set aside. Civil Cases Nos. 0288-J and 0289-J are hereby DISMISSED.

SO ORDERED.14

The RTC ruled that the evidence showed the better right of private respondent to possess Lot 1227. Private respondent’s position paper, affidavit and tax declaration supported her allegations. In addition, the commissioners’ report and sketch plan showed that indeed petitioners occupy Lot 1227. On the other hand, according to the RTC, the petitioners failed to present evidence which would show that they are entitled to possess the lot.

Based on the sketch plan, the RTC dismissed the cases against Gabasa and Amatorio since their houses occupy only a small area of Lot 1227. It declared that Gabasa and Amatorio believed in good faith that the whole area they occupied was part of the seashore.

The 19 petitioners, who were ordered to vacate the lot, filed a joint petition for review with the Court of Appeals. The appellate court denied the petition. Petitioners moved for reconsideration and filed an amended petition. The Court of Appeals, however, affirmed the factual findings and conclusions arrived at by the trial courts and denied the amended petition for lack of merit.15 It also denied the motion for reconsideration.

Petitioners are now before us, on a petition for review, alleging that:

The Honorable Court of Appeals, with due respect and deference, committed a reversible error in the interpretation/application of the law in the instant case and in the appreciation of the facts and evidence presented. The Court of Appeals gravely abused its discretion when it denied and dismissed the petition filed by the petitioners.16

After considering the parties’ submissions, we find three basic issues: (1) Did the MCTC err in taking jurisdiction over and deciding the cases? (2) Did the RTC err in sustaining the MCTC’s judgment? (3) Did the CA err in denying the petition for review filed by the 19 petitioners ordered to be ejected?

Petitioners insist that private respondent should have filed an action to recover possession de jure, not a mere complaint for ejectment, for two reasons. One, they possessed Lot 1227 in good faith for more than 30 years in the concept of owners. And two, there was no withholding of possession since private respondent was not in prior possession of the lot.

Page 10: Property Case

Private respondent states in her Comment before us that the allegations in her Complaints make out a clear case of unlawful detainer which is cognizable by the MCTC. We are in agreement with her stance. There was no error in the choice of the complainant’s remedy, a matter left to her determination as the suitor. And the complaint itself is defined by the allegations therein, not the allegations of the defendants.

At the outset, we note that petitioners question the MCTC’s jurisdiction yet they admit in their preliminary statement that the Complaints filed are indeed for unlawful detainer, and that the only issue to be determined is mere physical possession (possession de facto) and not juridical possession (possession de jure), much less ownership.17

While petitioners assert that this case involves only deprivation of possession, they confuse the remedy of an action for forcible entry with that of unlawful detainer. In unlawful detainer, prior physical possession by the plaintiff is not necessary. It is enough that plaintiff has a better right of possession. Actual, prior physical possession of a property by a party is indispensable only in forcible entry cases. In unlawful detainer cases, the defendant is necessarily in prior lawful possession of the property but his possession eventually becomes unlawful upon termination or expiration of his right to possess.18 Thus, the fact that petitioners are in possession of the lot does not automatically entitle them to remain in possession. And the issue of prior lawful possession by the defendants does not arise at all in a suit for unlawful detainer, simply because prior lawful possession by virtue of contract or other reasons is given or admitted. Unlike in forcible entry where defendants, by force, intimidation, threat, strategy or stealth, deprive the plaintiff or the prior physical possessor of possession. Here there is no evidence to show that petitioners entered the lot by any of these acts.

If only to stress the fundamental principles related to present controversy, jurisdiction over unlawful detainer suits is vested in municipal trial courts.19 And in ejectment cases, the jurisdiction of the court is determined by the allegations of the complaint.20

In this case for ejectment, private respondent’s allegations sufficiently present a case of unlawful detainer. She alleged that (1) she owns Lot 1227; (2) she tolerated petitioners to construct their houses thereon; (3) she withdrew her tolerance; and (4) petitioners refused to heed her demand to vacate the lot. The Complaints were also filed within one year from the date of her demand. The cause of action for unlawful detainer between the parties springs from the failure of petitioners to vacate the lot upon lawful demand of the private respondent. When they refused to vacate the lot after her demand, petitioners’ continued possession became unlawful. Her complaint for ejectment against respondent, to put it simply, is not without sufficient basis.

Petitioners’ contention that private respondent should have filed an action to recover possession de jure with the RTC is not supported by law or jurisprudence. The distinction between a summary action of ejectment and a plenary action for recovery of possession and/or ownership of the land is settled in our jurisprudence.

What really distinguishes an action for unlawful detainer from a possessory action (accion publiciana) and from a reinvindicatory action (accion reinvindicatoria) is that the first is limited to the question of possession de facto. An unlawful detainer suit (accion interdictal) together with forcible entry are the two forms of an ejectment suit that may be filed to recover possession of real property. Aside from the summary action of ejectment, accion publiciana or the plenary action to recover the right of possession and accion reinvindicatoria or the action to recover ownership which includes recovery of possession, make up the three kinds of actions to judicially recover possession.21

It is not up to defendants, now petitioners herein, to dictate upon plaintiff, now the private respondent, what her initial recourse should be. Her choice of an action for ejectment against so-called squatters is well within her rights.

Petitioners cite the case of Bayubay v. Court of Appeals,22 and argue that the MCTC’s decision was without jurisdictional or legal basis because the MCTC did not issue a preliminary conference order. They assert that the 10-day period to file position papers and affidavits only starts after the parties had received a preliminary conference order. They insist they were denied due process when the MCTC decided the cases based merely on private respondent’s Complaints and affidavit, without considering their Answers.

For her part, private respondent maintains that there was substantial compliance with the rules in the MCTC’s conduct of the preliminary conference, hence there was no violation of due process nor disregard of its proper jurisdiction.

Petitioners’ present contention was first raised only in their appeal to the RTC. Raising it before the appellate tribunal is barred by estoppel.23 They should have raised it in the proceedings before the MCTC. In our view, this issue is a mere afterthought, when the MCTC decided against them. Basic rules of fair play, justice and due process require that as a rule an issue cannot be raised by the petitioners for the first time on appeal.24

Besides, petitioners did not question initially the MCTC’s Order dated February 19, 1999, when they moved for an extension of time to file their position papers and affidavits. They wanted another 30 days on top of the 30 days set by the MCTC, which strictly should have been 10 days only. In this regard, petitioners could not claim that they were denied sufficient time to file their position papers and affidavits before the trial court. Further, they cannot validly invoke our ruling25 in Bayubay, for in that case there was no order at all terminating the preliminary conference and requiring the parties to submit position papers and affidavits.

We note with dismay petitioners’ insistence that we order the MCTC "to conduct the requisite preliminary conference." The summary character of ejectment suits will be disregarded if we allow petitioners to further delay this case by allowing a second preliminary conference. Ejectment by way of forcible entry and unlawful detainer cases are summary proceedings, designed to provide an expeditious means of protecting actual possession or the right to possession over the property involved. It is a timely procedure designed to remedy the delay in the resolution of such cases.26

Lastly, petitioners aver that private respondent failed to prove her allegation of ownership of Lot 1227 as it is only based on a tax declaration which is not an evidence of ownership. They also claim that their possession of the lot was not and could not be by mere tolerance. However, this is a factual matter best left to the trial courts.

What we have now is sufficient evidence showing that private respondent has a better right to possess Lot 1227. The commissioners’ report and sketch plan show that the 19 petitioners occupy the lot, which corroborate private respondent’s allegation and disprove petitioners’ defense that Lot 1227 is a shoreline; or that Lot 1227 is a social forest area. While not a conclusive evidence of ownership, private respondent’s tax declaration constitutes proof that she has a claim of title over the lot. It has been held that:

Although tax declarations or realty tax payment of property are not conclusive evidence of ownership, nevertheless, they are good indicia of possession in the concept of owner for no one in his right mind would be paying taxes for a property that is not in his actual or at least constructive possession. They constitute at least proof that the holder has a claim of title over the property. The voluntary declaration of a piece of property for taxation purposes manifests not only one’s sincere and honest desire to obtain title to the property and announces his adverse claim

Page 11: Property Case

against the State and all other interested parties, but also the intention to contribute needed revenues to the Government. Such an act strengthens one’s bona fide claim of acquisition of ownership.27

The lower courts did not err in adjudicating the issue of possession. Mere absence of title over the lot is not a ground for the courts to withhold relief from the parties in an ejectment case. Plainly stated, the trial court has validly exercised its jurisdiction over the ejectment cases below. The policy behind ejectment suits is to prevent breaches of the peace and criminal disorder, and to compel the party out of possession to respect and resort to the law alone to obtain what she claims is hers. The party deprived of possession must not take the law into his or her own hands.28 For their part, herein petitioners could not be barred from defending themselves before the court adequately, as a matter of law and right.

However, petitioners in their defense should show that they are entitled to possess Lot 1227. If they had any evidence to prove their defenses, they should have presented it to the MCTC with their position papers and affidavits. But they ignored the court’s order and missed the given opportunity to have their defenses heard, the very essence of due process.29 Their allegations were not only unsubstantiated but were also disproved by the plaintiff’s evidence.

In sum, we find no reversible error much less any grave abuse of discretion committed by the Court of Appeals. A person who occupies the land of another at the latter’s tolerance or permission, without any contract between them, is necessarily bound by an implied promise that he will vacate upon demand, failing which a summary action for ejectment is the proper remedy against him.30 His status is analogous to that of a lessee or tenant whose term of lease has expired but whose occupancy continued by tolerance of the owner. In such a case, the date of unlawful deprivation or withholding of possession is to be counted from the date of the demand to vacate.31

WHEREFORE, the instant petition is DENIED for lack of merit. The Decision of the Court of Appeals dated March 30, 2001 and its Resolution dated October 18, 2001 are AFFIRMED.

Costs against petitioners.

SO ORDERED.

Davide, Jr., C.J., (Chairman), Ynares-Santiago, Carpio, and Azcuna, JJ., concur.

Page 12: Property Case

Republic of the PhilippinesSUPREME COURT

SECOND DIVISION

G.R. No. 132197 August 16, 2005

ROSS RICA SALES CENTER, INC. and JUANITO KING & SONS, INC., Petitioners,

vs.SPOUSES GERRY ONG and ELIZABETH ONG, Respondent.

D E C I S I O N

Tinga, J.:

In a Decision1 dated 6 January 1998, the Former First Division of the Court of Appeals overturned the decisions of the Municipal Trial Court (MTC) and the Regional Trial Court (RTC) of Mandaue City, ruling instead that the MTC had no jurisdiction over the subject complaint for unlawful detainer. This petition for review prays for the reversal of the aforesaid Court of Appeals’ Decision.

The case originated from a complaint for ejectment filed by petitioners against respondents, docketed as Civil Case No. 2376, before the MTC of Mandaue City, Branch I. In the complaint, petitioners alleged the fact of their ownership of three (3) parcels of land covered by Transfer Certificates of Title (TCT) Nos. 36466, 36467 and 36468. Petitioners likewise acknowledged respondent Elizabeth Ong’s ownership of the lots previous to theirs. On 26 January 1995, Atty. Joseph M. Baduel, representing Mandaue Prime Estate Realty, wrote respondents informing them of its intent to use the lots and asking them to vacate within thirty (30) days from receipt of the letter. But respondents refused to vacate, thereby unlawfully withholding possession of said lots, so petitioners alleged.

Ross Rica Sales Center, Inc. and Juanito King and Sons, Inc. (petitioners) had acquired the lands from Mandaue Prime Estate Realty through a sale made on 23 March 1995. In turn, it appears that Mandaue Prime Estate Realty had acquired the properties from the respondents through a Deed of Absolute Sale dated 14 July 1994. However, this latter deed of sale and the transfers of title consequential thereto were subsequently sought to be annulled by respondents in a complaint filed on 13 February 1995 before the Mandaue RTC against Mandaue Prime Estate Realty.2 Per record, this case is still pending resolution.

Meanwhile, the MYC resolved the ejectment case on 24 April 1996, with the decision ordering respondents to vacate the premises in question and to peacefully turn over possession thereof to petitioners.

On appeal, the RTC rendered on 1 March 1997 a judgment affirming the MTC’s decision in its entirety.

On 8 May 1997, respondents filed a notice of appeal. However, on the following day, they filed a motion for reconsideration.

On 23 June 1997, the RTC issued an Order which concurrently gave due course to respondents’ notice of appeal filed on 8 May 1997; denied their motion for reconsideration dated 9 May 1997,3 and granted petitioners’ motion for immediate execution pending appeal.

In a Petition for Certiorari with Injunction filed with the Court of Appeals and treated as a Petition for Review, the appellate court ruled that the MTC had no jurisdiction over said case as there was no contract between the parties, express or implied, as would qualify the same as one for unlawful detainer. Thus, the assailed Orders of the MTC and RTC were set aside.

Petitioners then took this recourse via Petition for Review under Rule 45 of the Rules of Court. The principal issues raised before this Court are: (i) whether the RTC decision has already become final and executory at the time the petition for review was filed; (ii) whether the allegations in the complaint constitute a case for unlawful detainer properly cognizable by the MTC; and, (iii) whether petitioners, as registered owners, are entitled to the possession of the subject premises.

We resolve the first argument to be without merit.

The following sequence of events is undisputed:

(1) On 1 March 1997, the RTC rendered the questioned decision affirming the judgment of the MTC.

(2) On 28 April 1997, respondents received a copy of the aforementioned decision.

(3) On 8 May 1997, respondents filed a Notice of Appeal with the RTC.

(4) On 9 May 1997, respondents filed likewise with the RTC a Motion for Reconsideration of the aforementioned 1 March 1997 decision.

(5) On 23 June 1997, the RTC of Mandaue issued an Order denying respondents’ Motion for Reconsideration.

(6) On 9 July 1997, respondents received a copy of the aforementioned 23 June 1997 Order.

(7) On 24 July 1997, respondents filed with the Court of Appeals their motion for an additional period of ten (10) days within which to file their Petition for Review.

(8) On 30 July 1997, respondents filed with the Court of Appeals their Petition for Review.

Petitioners assert that the Petition for Review was filed beyond the fifteen (15)-day period for appeal. They theorize that the period started running on 28 April 1995, the date of receipt of the RTC decision, and ended on 13 May 1997. According to them, this reglementary period could not have been interrupted by the filing on 9 May 1997 of the Motion for Reconsideration because of the filing one day earlier of the Notice of Appeal. This Notice of Appeal dated 8 May 1997, albeit the wrong mode of appeal, expressly manifested their intention to file a petition for review to either the Court of Appeals or the Supreme Court.4

Petitioners further argue that respondents, after having filed the Notice of Appeal which was given due course by the RTC, cannot take an inconsistent stand such as filing a Motion for Reconsideration. Such filing, therefore, did not toll the fifteen (15)-day period which started running from the date of receipt of the RTC decision on 28 April 1997 and ended on 13 May 1997.

Respondents, in their Comment,5 submit that the filing of the Notice of Appeal dated 8 May 1997 was improper, and as such did not produce any legal effect. Therefore, the filing of the Motion for Reconsideration immediately on the following day cured this defect. The RTC refused to subscribe respondents’ position. It justified the denial of the Motion for Reconsideration on the ground that the respondents had already filed a Notice of Appeal. The Order dated 23 June 1997 stated:

On record is a Notice of Appeal by Certiorari filed by Defendants on May 8, 1997.

Likewise filed by Defendants on May 9, 1997 is a Motion for Reconsideration.

Considering the Notice of Appeal filed earlier which the court hereby approves, the Motion for Reconsideration is DENIED.

The Motion for Immediate Execution Pending Appeal being meritorious, is GRANTED.6 (Emphasis in the original.)

Page 13: Property Case

Strangely enough, the Court of Appeals passed no comment on this point when it took cognizance of respondents’ position and reversed the RTC. But does this necessarily mean that the RTC was correct when it declared that the Motion for Reconsideration was barred by the filing of the Notice of Appeal, no matter how erroneous the latter mode was?

Rule 42 governs the mode of appeal applicable in this case. Sec. 1 provides:

Section 1. How appeal taken; time for filing. -- A party desiring to appeal from a decision of the RTC rendered in the exercise of its appellate jurisdiction may file a verified petition for review with the Court of Appeals, paying at the same time to the clerk of said court the corresponding docket and other lawful fees, depositing the amount of P500.00 for costs, and furnishing the Regional Trial Court and the adverse party with a copy of the petition. The petition shall be filed and served within fifteen (15) days from notice of the decision sought to be reviewed or of the denial of petitioner’s motion for new trial or reconsideration filed in due time after judgment. Upon proper motion and the payment of the full amount of the docket and other lawful fees and the deposit for costs before the expiration of the reglementary period, the Court of Appeals may grant an additional period of fifteen (15) days only within which to file the petition for review. No further extension shall be granted except for the most compelling reason and in no case to exceed fifteen (15) days.

Since the unlawful detainer case was filed with the MTC and affirmed by the RTC, petitioners should have filed a Petition for Review with the Court of Appeals and not a Notice of Appeal with the RTC. However, we consider this to have been remedied by the timely filing of the Motion for Reconsideration on the following day. Section 3, Rule 50 of the Rules of Court allows the withdrawal of appeal at any time, as a matter of right, before the filing of the appellee’s brief. Applying this rule contextually, the filing of the Motion for Reconsideration may be deemed as an effective withdrawal of the defective Notice of Appeal.

Perforce, the period of appeal was tolled by the Motion for Reconsideration and started to run again from the receipt of the order denying the Motion for Reconsideration. A Motion for Additional Time to File the Petition was likewise filed with the Court of Appeals. Counting fifteen (15) days from receipt of the denial of the Motion for Reconsideration and the ten (10)-day request for additional period, it is clear that respondents filed their Petition for Review on time.

Petitioners invoke to the ruling in People v. De la Cruz7 that once a notice of appeal is filed, it cannot be validly withdrawn to give way to a motion for reconsideration. The factual circumstances in the two cases are different.

De la Cruz is a criminal case, governed by criminal procedure. Section 3, Rule 122 of the Rules of Court provides that the proper mode of appeal from a decision of the RTC is a notice of appeal and an appeal is deemed perfected upon filing of the notice of appeal.

In the case at bar, a petition for review before the Court of Appeals is the proper mode of appeal from a decision of the RTC. Since the filing of the notice of appeal is erroneous, it is considered as if no appeal was interposed.

Now on the second and more important issue raised by petitioners: whether the Complaint satisfies the jurisdictional requirements for a case of unlawful detainer properly cognizable by the MTC.

The MTC considered itself as having jurisdiction over the ejectment complaint and disposed of the same in favor of petitioners. Said ruling was affirmed by the RTC. The Court of Appeals reversed the lower courts and found the complaint to be one not for unlawful detainer based on two (2) grounds, namely: that the allegations fail to show that petitioners were deprived of possession by force, intimidation, threat, strategy or stealth; and that there is no contract, express or implied, between the parties as would qualify the case as one of unlawful detainer.

We disagree with the Court of Appeals.

The complaint for unlawful detainer contained the following material allegations:

. . . .

3. That plaintiffs are the owners of Lot No. 2, which is covered by T.C.T. No. 36466 of the Register of Deeds of Mandaue City, Lot No. 1-A which is covered by T.C.T. No. 36467 of the Register of Deeds of Mandaue City and Lot No. 86-A which is covered by T.C.T. No. 36468 of the Register of Deeds of Mandaue City, all situated in the City of Mandaue. Copies of said Transfer Certificate of Titles are hereto attached as Annexes "A", "B", and "C" respectively and made an integral part hereof;

4. That defendant Elizabeth Ong is the previous registered owner of said lots;

5. That as the previous registered owner of said lots, defendant Elizabeth Ong and her husband and co-defendant Jerry Ong have been living in the house constructed on said lots;

6. That on May 6, 1995, plaintiffs, through the undersigned counsel, wrote defendants a letter informing them or their intent to use said lots and demanded of them to vacate said lots within 30 days from receipt of said letter. Copy of said letter is hereto attached as Annex "D" and made an integral part thereof;

7. That despite demand to vacate, the defendants have refused and still refuse to vacate said lots, thus, unlawfully withholding possession of said lots from plaintiffs and depriving plaintiffs of the use of their lots;

8. That in unlawfully withholding the possession of said lots from the plaintiffs, plaintiffs have suffered damages in the form of unearned rentals in the amount of P10,000.00 a month

. . . .8

Well-settled is the rule that what determines the nature of an action as well as which court has jurisdiction over it are the allegations of the complaint and the character of the relief sought.9

Respondents contend that the complaint did not allege that petitioners’ possession was originally lawful but had ceased to be so due to the expiration of the right to possess by virtue of any express or implied contract.

The emphasis placed by the Court of Appeals on the presence of a contract as a requisite to qualify the case as one of unlawful detainer contradicts the various jurisprudence dealing on the matter.

In Javelosa v. Court of the Appeals,10 it was held that the allegation in the complaint that there was unlawful withholding of possession is sufficient to make out a case for unlawful detainer. It is equally settled that in an action for unlawful detainer, an allegation that the defendant is unlawfully withholding possession from the plaintiff is deemed sufficient, without necessarily employing the terminology of the law.11

Hence, the phrase "unlawful withholding" has been held to imply possession on the part of defendant, which was legal in the beginning, having no other source than a contract, express or implied, and which later expired as a right and is being withheld by defendant.12 In Rosanna B. Barba v. Court of Appeals,13 we held that a simple allegation

that the defendant is unlawfully withholding possession from plaintiff is sufficient.

Based on this premise, the allegation in the Complaint that:

. . . . despite demand to vacate, the defendants have refused and still refuse to vacate said lots, thus, unlawfully withholding possession of said lots from plaintiffs and depriving plaintiffs of the use of their lots;14

Page 14: Property Case

is already sufficient to constitute an unlawful detainer case.

In the subject complaint, petitioners alleged that they are the registered owners of the lots covered by TCT Nos. 36466, 36467 and 36468. By their implied tolerance, they have allowed respondents, the former owners of the properties, to remain therein. Nonetheless, they eventually sent a letter to respondents asking that the latter vacate the said lots. Respondents refused, thereby depriving petitioners of possession of the lots. Clearly, the complaint establishes the basic elements of an unlawful detainer case, certainly sufficient for the purpose of vesting jurisdiction over it in the MTC.

Respondents would like to capitalize on the requisites as cited in the case of Raymundo dela Paz v. Panis.15 But the citation is a mere reiteration of Sec. 1, Rule 7016 of the Rules of Court. The case doesid not provide for rigid standards in the drafting of the ejectment complaint. The case of Co Tiamco v. Diaz17 justifies a more liberal approach, thus:

. . . The principle underlying the brevity and simplicity of pleadings in forcible entry and unlawful detainer cases rests upon considerations of public policy. Cases of forcible entry and detainer are summary in nature, for they involve perturbation of social order which must be restored as promptly as possible and, accordingly, technicalities or details of procedure should be carefully avoided.18

Moreover, petitioners fail to mention any of the incidents of the pending case involving the annulment of deed of sale and title over said property. Petitioners know better than to question this in an ejectment proceeding, which brings us to the nature of the action in this case.

Respondents insist that the RTC, and not the MTC, had jurisdiction over the action, it being an accion reivindicatoria according to them, on the ground that petitioners were constantly claiming ownership over the lands in the guise of filing an action for ejectment. In their Comment,19 respondents maintain that they occupy the subject lots as the legal owners. Petitioners, on the other hand, are seeking recovery of possession under a claim of ownership which is tantamount to recovery of possession based on alleged title to the lands, and therefore is within the original jurisdiction of the RTC, so respondents conclude.

This contention is not tenable.

The issue involved in accion reivindicatoria is the recovery of ownership of real property. This differs from accion publiciana where the issue is the better right of possession or possession de jure, and accion interdictal where the issue is material possession or possession de facto. In an action for unlawful detainer, the question of possession is primordial while the issue of ownership is generally unessential.20

Neither the allegation in petitioners’ complaint for ejectment nor the defenses thereto raised by respondents sufficiently convert this case into an accion reivindicatoria which is beyond the province of the MTC to decide. Petitioners did not institute the complaint for ejectment as a means of claiming or obtaining ownership of the properties. The acknowledgment in their pleadings of the fact of prior ownership by respondents does not constitute a recognition of respondents’ present ownership. This is meant only to establish one of the necessary elements for a case of unlawful detainer, specifically the unlawful withholding of possession. Petitioners, in all their pleadings, only sought to recover physical possession of the subject property. The mere fact that they claim ownership over the parcels of land as well did not deprive the MTC of jurisdiction to try the ejectment case.

Even if respondents claim ownership as a defense to the complaint for ejectment, the conclusion would be the same for mere assertion of ownership by the defendant in an ejectment case will not therefore oust the municipal court of its summary jurisdiction.21 This Court in Ganadin

v. Ramos22 stated that if what is prayed for is ejectment or recovery of possession, it does not matter if ownership is claimed by either party. Therefore, the pending actions for declaration of nullity of deed of sale and Transfer Certificates of Title and quieting of title in Civil Case No. MAN-2356 will not abate the ejectment case.

In Drilon v. Gaurana,23 this Court ruled that the filing of an action for reconveyance of title over the same property or for annulment of the deed of sale over the land does not divest the MTC of its jurisdiction to try the forcible entry or unlawful detainer case before it, the rationale being that, while there may be identity of parties and subject matter in the forcible entry case and the suit for annulment of title and/or reconveyance, the rights asserted and the relief prayed for are not the same.24

In Oronce v. Court of Appeals,25 this Court held that the fact that respondents had previously filed a separate action for the reformation of a deed of absolute sale into one of pacto de retro sale or equitable mortgage in the same

Court of First Instance is not a valid reason to frustrate the summary remedy of ejectment afforded by law to the plaintiff. Consequently, an adjudication made in an ejectment proceeding regarding the issue of ownership should be regarded as merely provisional and, therefore, would not bar or prejudice an action between the same parties involving title to the land. The foregoing doctrine is a necessary consequence of the nature of forcible entry and unlawful detainer cases where the only issue to be settled is the physical or material possession over the real property, that is, possession de facto and not possession de jure.

The Court reiterated this in the case of Tecson v. Gutierrez26 when it ruled:

We must stress, however, that before us is only the initial determination of ownership over the lot in dispute, for the purpose of settling the issue of possession, although the issue of ownership is inseparably linked thereto. As such, the lower court's adjudication of ownership in the ejectment case is merely provisional, and our affirmance of the trial courts' decisions as well, would not bar or prejudice an action between the same parties involving title to the property, if and when such action is brought seasonably before the proper forum.

The long settled rule is that the issue of ownership cannot be subject of a collateral attack.

In Apostol v. Court of Appeals,27 this Court had the occasion to clarify this:

. . . Under Section 48 of Presidential Decree No. 1529, a certificate of title shall not be subject to collateral attack. It cannot be altered, modified or cancelled, except in a direct proceeding for that purpose in accordance with law. The issue of the validity of the title of the respondents can only be assailed in an action expressly instituted for that purpose. Whether or not the petitioners have the right to claim ownership over the property is beyond the power of the court a quo to determine in an action for unlawful detainer.28

With the conclusion of the second issue in favor of petitioners, there is no need to discuss the third assignment of error which is related to the second issue.

WHEREFORE, the Petition is GRANTED. The Decision of the Court of Appeals dated 6 January 1998 is REVERSED and SET ASIDE and the Decision dated 24

April 1996 of the Municipal Trial Court of Mandaue City REINSTATED and AFFIRMED. Costs against respondents.

SO ORDERED.

DANTE O. TINGA Associate Justice

Page 15: Property Case

Republic of the PhilippinesSUPREME COURT

FIRST DIVISION

G.R. No. 165177 August 25, 2005

LILIA V. PERALTA-LABRADOR, Petitioners, vs.SILVERIO BUGARIN, substituted by his widow, CONSOLACION BUGARIN,1 Respondent.

DECISION

YNARES-SANTIAGO, J.:

Challenged in this petition for review on certiorari is the March 12, 2004 decision2 of the Court of Appeals in CA-G.R. SP No. 57475, which affirmed with modification the January 26, 2000 judgment3 of the Regional Trial Court (RTC) of Iba, Zambales, Branch 71, in Civil Case No. RTC-1590-I, which in turn affirmed the decision4 dated May 16, 1999 of the Municipal Trial Court (MTC) of San Felipe, Zambales, in Civil Case No. 328, and its September 6, 2004 resolution5 denying reconsideration thereof.

On January 18, 1996, petitioner Lilia V. Peralta-Labrador filed a case for "Recovery of Possession and Ownership," docketed as Civil Case No. 328, with the MTC of San Felipe, Zambales. She alleged that she is the owner of Cadastral Lot No. 2650, with an area of 400 sq. m. located at Sitio Caarosipan, Barangay Manglicmot, San Felipe, Zambales, having purchased the same in 1976 from spouses Artemio and Angela Pronto. In 1977, she was issued Tax Declaration No. 10462 and paid the taxes due thereon.6

In 1990, the Department of Public Works and Highways constructed a road which traversed Cadastral Lot No. 2650 thereby separating 108 sq. m. from the rest of petitioner’s lot, for which she was issued Tax Declaration No. 02-2460R in 1991.7

Sometime in 1994, respondent Silverio Bugarin forcibly took possession of the 108 sq. m. lot and refused to vacate the same despite the pleas of petitioner. Hence, on January 18, 1996, she instituted a complaint for recovery of possession and ownership against respondent.

In his Answer with Counterclaims,8 respondent contended that the area claimed by petitioner is included in the 4,473 square meter lot, covered by the Original Certificate of Title (OCT) No. P-13011; and that he has been in continuous possession and occupation thereof since 1955. In his Amended Answer with Counterclaim,9 however, respondent failed to allege that the questioned lot is covered by the OCT No. P-13011, and instead asserted that he planted fruit bearing trees in the property. Respondent further pleaded the defenses of lack of cause of action and prescription.

On May 16, 1999, the court a quo ruled in favor of respondent declaring him as the owner of the controverted lot on the basis of the OCT No. P-13011. The complaint was dismissed for failure of petitioner to prove prior physical possession and ownership thereof. The dispositive portion thereof, reads:

WHEREFORE, all the foregoing premises considered and for failure on the part of the plaintiff to establish the preponderance of evidence of prior actual physical possession and present title over the lot in her favor, let the instant case be ordered DISMISSED, and the defendant be awarded the rightful possession and ownership of the same and the plaintiff is hereby ordered to pay FIFTEEN THOUSAND (P15,000.00) PESOS as reasonable Attorney’s fee and FIVE THOUSAND (P5,000.00) PESOS as appearance fee plus costs.

SO ORDERED.10

The RTC affirmed the assailed decision,11 hence petitioner filed a petition for review before the Court of Appeals which was however denied for insufficiency of evidence to prove ownership or prior actual physical possession. The appellate court deleted the monetary awards in favor of respondent as well as the declaration of the MTC that respondent is the owner of the questioned lot on the ground that the OCT No. P-13011, relied upon by said court was not formally offered in evidence, hence, cannot be considered by the court. The decretal portion thereof, states:

WHEREFORE, in view of the foregoing discussion, the instant petition is hereby PARTIALLY GRANTED. The assailed Decision dated January 26, 2000, in Civil Case No. RTC 1590 I of the Regional Trial Court (RTC), Branch 71, Iba, Zambales, and Decision dated May 16, 1999, in Civil Case No. 328 of the Municipal Trial Court of San Felipe, Zambales are MODIFIED by deleting the declaration of ownership as to the disputed 108 square meters and the monetary award in favor of respondent Silverio Bugarin. However, the dismissal of the complaint is AFFIRMED.

SO ORDERED.12

The motion for reconsideration filed by petitioner was denied. Hence the instant petition.

Pertinent portion of Section 1, Rule 70 of the Revised Rules of Civil Procedure, provides:

SECTION 1. Who may institute proceedings, and when. – … a person deprived of the possession of any land or building by force, intimidation, threat, strategy, or stealth, … may at any time within one (1) year after such unlawful deprivation or withholding of possession, bring an action in the proper Municipal Trial Court against the person or persons unlawfully withholding or depriving of possession, or any person or persons claiming under them, for the restitution of such possession, together with the damages and costs. (Emphasis supplied)

In Lopez v. David Jr.,13 it was held that an action for forcible entry is a quieting process and the one year time bar for filing a suit is in pursuance of the summary nature of the action. Thus, we have nullified proceedings in the MTCs when it improperly assumed jurisdiction of a case in which the unlawful deprivation or withholding of possession had exceeded one year. After the lapse of the one year period, the suit must be commenced in the RTC via an accion publiciana, a suit for recovery of the right to possess. It is an ordinary civil proceeding to determine the better right of possession of realty independently of title. It also refers to an ejectment suit filed after the expiration of one year from the accrual of the cause of action or from the unlawful withholding of possession of the realty independently of title. Likewise, the case may be instituted before the same court as an accion reivindicatoria, which is an action to recover ownership as well as possession.14

Corrollarily, jurisdiction of a court is determined by the allegations of the complaint. Thus, in ascertaining whether or not the action falls within the exclusive jurisdiction of the inferior courts, the averments of the complaint and the character of the relief sought are to be examined.15

In the instant case, petitioner’s complaint alleges that:

2. That plaintiff is the owner of a parcel of land denominated as Cadastral lot No. 2650, San Felipe Cadastre, situated at sitio Caarosipan, Barangay Manglicmot, San Felipe, Zambales which she bought in 1976 from Spouses Artemio Pronto and Angela Merano when she was still a widow, with the following boundaries: North, Alipio Abad, East, Antonio Cueva, South, Juan Borja, and West, Old Provincial Road, containing an area of 108 square meters, declared under Tax Declaration No. 002-1860R and assessed at P1,120.00;

Page 16: Property Case

3. That plaintiff has been in open, continuous, exclusive and adverse as well as notorious possession of the said lot and in the concept of an owner since she [acquired] it in 1976 until the time when defendant took possession forcibly, two years ago;

4. That in or before 1990 the land was traversed by a new National Highway and the land was segregated from a bigger portion of the land, the western portion is now the land in question and since the new provincial road which traversed the whole land of the plaintiff, the old highway which is west of Lot 2650 shall belong to the plaintiff in compensation of the portion of her lot traversed by the new highway, said old highway is also taken by defendant unlawfully;16

It is clear that petitioner’s averment make out a case for forcible entry because she alleged prior physical possession of the subject lot way back in 1976, and the forcible entry thereon by respondent. Considering her allegation that the unlawful possession of respondent occurred two years17 prior to the filing of the complaint on January 18, 1996, the cause of action for forcible entry has prescribed and the MTC had no jurisdiction to entertain the case. Petitioner’s complaint therefore should have been filed with the proper RTC.

It is settled that jurisdiction over the subject matter cannot be waived by the parties or cured by their silence, acquiescence or even express consent.18 Hence, the failure of respondent to insist on the defenses of lack of cause of action and prescription stated in his Amended Answer with Counterclaim will not vest the MTC with jurisdiction over the case.

On this point, the Court held in Bongato v. Malvar19 that:

It is wise to be reminded that forcible entry is a quieting process, and that the restrictive time bar is prescribed to complement the summary nature of such process. Indeed, the one-year period within which to bring an action for forcible entry is generally counted from the date of actual entry to the land. However, when entry is made through stealth, then the one-year period is counted from the time the plaintiff learned about it. After the lapse of the one-year period, the party dispossessed of a parcel of land may file either an accion publiciana, which is a plenary action to recover the right of possession; or an accion reivindicatoria, which is an action to recover ownership as well as possession.

On the basis of the foregoing facts, it is clear that the cause of action for forcible entry filed by respondents had already prescribed when they filed the Complaint for ejectment on July 10, 1992. Hence, even if Severo Malvar may be the owner of the land, possession thereof cannot be wrested through a summary action for ejectment of petitioner, who had been occupying it for more than one (1) year. Respondents should have presented their suit before the RTC in an accion publiciana or an accion reivindicatoria, not before the MTCC in summary proceedings for forcible entry. Their cause of action for forcible entry had prescribed already, and the MTCC had no more jurisdiction to hear and decide it.

...

Further, a court’s lack of jurisdiction over the subject matter cannot be waived by the parties or cured by their silence, acquiescence or even express consent. A party may assail the jurisdiction of the court over the action at any stage of the proceedings and even on appeal. That the MTCC can take cognizance of a motion to dismiss on the ground of lack of jurisdiction, even if an answer has been belatedly filed we likewise held in Bayog v. Natino[.]

Moreover, even if the MTC has jurisdiction over the subject matter, the complaint should still be dismissed because petitioner failed to prove that the controverted 108 sq. m. lot is part of Cadastral Lot No. 2650. Petitioner admitted that she has never seen the Cadastral Map of San Felipe, Zambales, and relied only on the Survey Notification Card20 from the Bureau of Lands,21 with a sketch of Cadastral Lot

No. 2650. Said card, however, does not reflect the 108 sq. m. lot subject of this case. Neither did petitioner cause the survey of Cadastral Lot No. 2650 after the construction of a new road to prove that the segregated portion on the western side is part thereof. Ei incumbit probotio qui dicit, non qui negat. He who asserts, not he who denies, must prove.22 Failing to discharge this burden, the dismissal of the complaint is proper.

In the same vein, ownership of the lot in question cannot be awarded to respondent considering that OCT No. P-13011,23 and the Survey Plan24 were not formally offered in evidence. While the issue of ownership may be passed upon in ejectment cases for the sole purpose of determining the nature of possession,25 no evidence conclusively show that the lot in question is covered by said OCT No. P-13011 or any other title of respondent.

WHEREFORE, the May 16, 1999 decision of the Municipal Trial Court of San Felipe, Zambales, the January 26, 2000 decision of the Regional Trial Court, Branch 71, Iba, Zambales, and the March 12, 2004 decision of the Court of Appeals, are ANNULLED and SET ASIDE for lack of jurisdiction. The complaint in Civil Case No. 328 is DISMISSED.

SO ORDERED.

CONSUELO YNARES-SANTIAGO

Page 17: Property Case

Republic of the PhilippinesSUPREME COURT

Manila

SECOND DIVISION

G.R. No. 127382             August 17, 2004

DR. JESUS SERIÑA and ENRIQUETA SERIÑA (deceased), represented by DR. JESUS SERIÑA, JR., ANTONIO SERIÑA, VIOLETA SERIÑA TAN, REYNALDO SERIÑA and EMMANUEL SERIÑA, petitioners, vs.VICTOR CABALLERO, TEODORO DONELA, OLIVER DONELA, COURT OF APPEALS, and THE HONORABLE REGIONAL TRIAL COURT, BRANCH 20, MISAMIS ORIENTAL, respondents.

D E C I S I O N

CALLEJO, SR., J.:

Before us is a petition for review on certiorari of the Decision1 of the Court of Appeals (CA) dated August 23, 1996, affirming the dismissal of the complaint for quieting of title, recovery of possession, and damages by the Regional Trial Court (RTC) of Misamis Oriental, Cagayan de Oro City, in Civil Case No. 8716.

The Antecedents

On August 11, 1982, Dr. Jesus Seriña and his wife, Enriqueta Seriña filed a Complaint for quieting of title, recovery of possession, and damages with a prayer for a writ of preliminary mandatory injunction against respondents Victor Caballero and his tenants, Teodoro Donela and Oliver Donela. When Dr. Seriña died on August 6, 1983, he was substituted by his children, petitioners Jesus, Jr., Antonio, Violeta, Reynaldo and Emmanuel.2

The petitioners alleged in their complaint that they are the absolute owners and have been in actual and constructive possession for thirty-five (35) years of a parcel of land described as follows:

Lot No. 3533-A, Cad-237, Cagayan Cadastre

Tax Declaration No. 02161

Location - Mantadiao, Opol,

Misamis Oriental

Area - 2.5000 has.

Boundaries:

North - Alejo Seriña

South - T. Sabornido

East - A. Seriña & T. Sabornido

West - F. Caballero3

The petitioners averred that sometime in March 1982, they discovered that respondent Caballero was claiming ownership over the said land and offering it for sale or mortgage to third parties. They also discovered that the respondents Donelas were occupying the land as tenants and caretakers of the land. 4

The petitioners claimed that their father, Dr. Seriña, bought the land from Lucia Vda. de Marbella who inherited it from her father, Ramon Neri.5 They presented a Deed of Sale6 dated August 23, 1947 showing that Dr. Seriña bought 5 hectares of ricefield, bounded on the North by Raymundo Seriña, on the East by Teofilo Saburnido, on the South by Obdelio Caballero, on the West by Obdullo Caballero, from Lucia Vda. de Marbella. Dr. Seriña was issued Tax Declaration No. 4029 allegedly for the said property. As indicated in the tax declaration and subsequent tax declarations issued in the name of Dr. Seriña, they were issued for Cadastral Lot No. 3533 and covered a 2.5-hectare ricefield with the same boundary owners as those in the complaint.7 The petitioners also averred that they regularly paid taxes thereon since 1947 up to the present.8

In his answer, respondent Caballero alleged that he was the lawful owner, and had been in actual physical possession of the disputed land since time immemorial. He averred that the disputed land is part of Cadastral Lot No. 3533, C-7 of the Cagayan Cadastre and originally owned by his grandfather, Eustaquio Caballero.9

The respondents averred that Eustaquio Caballero declared the entire parcel of land for tax purposes even before the war. Tax Declaration No. 2442 was issued in lieu of the records that were destroyed during the war.

This tax declaration indicated that the 119,490 square-meter parcel of land was located at Pontacon, Iponan, Cagayan de Oro City, bounded on North by Rustico Dablio, on the East by J. Seriña and T. Saburnido, on the South by Victor Obsioma, and on the West by Victorino Caballero.10

Emiliana Ibarat, respondent Caballero’s sister, testified that when Eustaquio Caballero died in 1944, the land was divided among his three children, Vicenta, Benita and Victorino, the father of respondent Caballero. Lot A, with an area of 39,625 square meters, was given to Victorino, which was later inherited by the respondent. Lot B, with an area of 71, 450 square meters, was given to Benita; and Lot C, with only 7,938 square meters was given to Vicenta. Lots B and C were, thereafter, sold to one Gaga Yasay. Because of the trouble between the petitioners and the respondents, Yasay agreed to buy only a portion of Lot A.11

The land was surveyed during the trial and it was determined that it now consisted of only 23,373 square meters,12 and not 25,000 square meters as claimed by the petitioners. Gliceria Legaspi, respondent Caballero’s other sister, also testified that the disputed land was now bounded on the North by Seriña and Nangcas, on the East by Teofilo Saburnido, on the South by Gaga Yasay, and on the West by Nangcas.13

The RTC rendered judgment14 on January 21, 1992, dismissing the complaint, and upholding the right of the respondents over the land. The dispositive portion reads:

WHEREFORE, judgment is hereby rendered in favor of the defendant Victor Caballero and against the plaintiffs herein, to wit:

1. Ordering the dismissal of the complaint with costs.

2. Ordering the defendant Victor Caballero as the absolute and lawful owner and possessor of the land in question.

3. Ordering the plaintiffs, their heirs, lawyers, servants or privies not to disturb or molest the possession and ownership of Victor Caballero over the land in question.

4. Ordering the plaintiffs to pay to defendant Victor Caballero, jointly and severally the sum of FIVE THOUSAND (P5,000.00) pesos for expenses of litigation, and THREE THOUSAND (P3,000.00) pesos for and as attorney's fees having been compelled to retain the services of counsel to protect his interest herein.

Page 18: Property Case

SO ORDERED.15

The trial court ruled that it was not clearly shown that the land bought by Dr. Seriña from Lucia Vda. de Marbella was the same land owned by Victor Caballero, and that the petitioners failed to show that Lucia Vda. de Marbella bought the land from Eustaquio Caballero, the original owner and cadastral claimant of the land. It also noted that the deed of sale between Lucia Vda. de Marbella and Dr. Seriña showed that the land had an area of 5 hectares, whereas, the petitioners only claimed 2.5 hectares. Furthermore, the boundaries of the land stated in the complaint did not coincide with what was stated in the Deed of Sale, or in Tax Declaration No. 2442 in the name of Eustaquio Caballero. The trial court ruled that the petitioners failed to explain these discrepancies, and that there was no showing that Tax Declaration No. 2442 was cancelled by Tax Declaration No. 4029 in the name of Dr. Seriña. The trial court interpreted this to mean that Eustaquio Caballero's right as owner of the land remained.

Dissatisfied, the petitioners appealed the case to the CA, which rendered a Decision16 affirming in toto the decision of the RTC. The petitioners filed a Motion for Reconsideration on September 30, 1996.17 The CA denied the motion.18

Hence, the instant petition.

The petitioners assign the following errors:

1. THAT IT IS ERROR FOR THE HONORABLE COURT OF APPEALS TO UPHOLD THE HONORABLE RTC ON THE ISSUE THAT THE ALLEGED IDENTITY OF THE LAND IN LITIGATION IS UNESTABLISHED BETWEEN THE PARTIES-LITIGANTS.

2. THAT IT IS ERROR FOR THE HONORABLE COURT OF APPEALS TO FAIL TO APPRECIATE THE 35-YEAR ACQUISITIVE PRESCRIPTION IN FAVOR OF THE PLAINTIFFS-APPELLANTS.19

The issues in this petition are, therefore, the following: (1) whether the petitioners were able to establish the identity of the land being claimed by them; and (2) whether acquisitive prescription should be appreciated in favor of the petitioners.

The Ruling of the Court

The first issue deals clearly with a question of fact which is beyond the province of this Court in a petition for review on certiorari. Well-entrenched is the rule that the Court's jurisdiction in a petition for review is limited to reviewing or revising errors of law allegedly committed by the appellate court. Factual findings of the Court of Appeals are conclusive on the parties and not reviewable by this Court—and they carry even more weight when the Court of Appeals affirms the factual findings of the trial court.20 The exceptions to this rule are the following:

(1) when the conclusion is a finding grounded entirely on speculations, surmises or conjectures; (2) when the inference made is manifestly mistaken, absurd or impossible; (3) when there is grave abuse of discretion; (4) when the judgment is based on misapprehension of facts; (5) when the findings of facts are conflicting; (6) when the Court of Appeals, in making its findings, went beyond the issues of the case and the same is contrary to the admissions of both appellant and appellee; (7) when the findings of the Court of Appeals are contrary to those of the trial court; (8) when the findings of fact are conclusions without citation of specific evidence on which they are based; (9) when the Court of Appeals manifestly overlooked certain relevant facts not disputed by the parties, which, if properly considered, would justify a different conclusion; and (10) when the findings of fact of the Court of Appeals are premised on the absence of evidence and are contradicted by the evidence on record.21

We find no cogent reason to reverse the findings of the CA. None of the aforementioned exceptions is present in this case. The CA was correct in concluding

that the petitioners failed to establish that the parcel of land in the possession of the respondents is the same as that subject of their complaint.

The CA noted that the land subject of the complaint has boundaries different from the land in possession of the respondents. In fact, the land described in the complaint appears to be different from the land described in the Deed of Sale which the petitioners invoke as the basis of their ownership.

First. The petitioners alleged in their complaint that the boundaries of their property are as follows:

North - Alejo Seriña

South - T. Sabornido

East - A. Seriña & T. Sabornido

West - F. Caballero22

On the other hand, the Deed of Sale provides that the property sold to them has the following boundaries:

North - Raymundo Seriña

South - Obdullo Caballero

East - Teofilo Saburnido

West - Obdullo Caballero23

Second. The complaint24 of the petitioners states that the property they are claiming has an area of 2.5 hectares. On the other hand, the Deed of Sale25 provides that the subject property has an area of 5 hectares.

Third. The complaint alleged that the property is located in "Mantadiao, Opol, Misamis Oriental,"26 while the Deed of Sale shows that the property purchased is located in "Puntakon, Igpit, Cagayan Or. Misamis."27

We agree with the CA that there was no showing that Tax Declaration No. 2442 in the name of Eustaquio Caballero was cancelled. Absent any specific statement therein to that effect, it cannot be presumed that Tax Declaration No. 4029 in the name of Dr. Seriña cancelled Tax Declaration No. 2442.

Moreover, the land covered by Tax Declaration No. 2442 is different from that covered by Tax Declaration No. 4029 for the following reasons:

The boundary owners of the land as indicated in Tax Declaration No. 2442 differ from those stated in Tax Declaration No. 4029. The boundary owners as indicated in Tax Declaration No. 2442 are as follows:

North - Rustico Dablio

South -Victor Obsioma

East - J. Seriña & T. Saburnido

West - Victorino Caballero28

Under Tax Declaration No. 4029, on the other hand, the boundary owners are as follows:

North - Alejo Seriña

South - Teofilo Saburnido

East - A. Seriña [and] T. Saburnido

West - Eustaquio Caballero29

Page 19: Property Case

Moreover, Tax Declaration No. 2442 covers an area of 119,490 square meters30 while Tax Declaration No. 4029 covers only 25,000 square meters or 2.5 hectares.31

The petitioners argue that the Deed of Sale and Tax Declaration No. 4029 should not be compared to Tax Declaration No. 2442 and the Technical Description of Cadastral Lot No. 3533 because the former refers only to a portion of the area referred to by the latter.32 While the petitioners are correct on this point, such mistake would still not justify a different conclusion. The fact remains that the documentary and testimonial evidence presented by the petitioners did not prove the identity of the land being claimed. The petitioners did not present evidence to prove that the land registered in the name of Eustaquio Caballero was sold to Lucia Vda. de Marbella or her predecessor-in-interest from whom they purchased the land subject of their complaint.

The failure to establish the identity of the land is obviously fatal to the petitioners’ case. In Beo vs. Court of Appeals,33 a case which also involves an action for possession and quieting of title, the Court had the occasion to state:

…[B]ecause petitioners failed to explain the discrepancy or present other evidence to prove with certainty the location and area of the land they seek to recover, respondent court correctly applied the invariable rule that a person who claims ownership of real property is duty-bound to clearly identify the land being claimed, in accordance with the title on which he anchors his right of ownership. When the record does not show that the land subject matter of the action for recovery of possession has been exactly determined, such action cannot prosper, as in the case of petitioners. In sum, proof of ownership coupled with identity of the land is the basic rule.

Corollarily, the rule is likewise well-settled that in order that an action for recovery of possession may prosper, it is indispensable that he who brings the action fully proves not only his ownership but also the identity of the property claimed, by describing the location, area and boundaries thereof. As the appellate court succinctly stated, he who claims to have a better right to the property must clearly show that the land possessed by the other party is the very land that belongs to him.34

On the second issue, the CA ruled that inasmuch as the petitioners failed to establish that the parcel of land in possession of the respondents is the same as the subject of their complaint, their claim of acquisitive prescription is clearly untenable.

The petitioners argue that they would not have regularly paid taxes on the land since 1947 had they not believed that they owned the same.35 The respondents, for their part, aver that the petitioners were only able to prove seven (7) years of actual possession of the land through cultivation by their tenants. They argue that such seven-year period of cultivation cannot be considered in the petitioners’ favor, since the witness who testified on this fact did not personally know the boundaries of the land cultivated, or whether it was the same land bought by Dr. Seriña. The respondents contend that acquisitive prescription applies only when there is no dispute as to the identity of the property.36

We agree with the respondents. Since the property has not been clearly identified by the petitioners, their claim of acquisitive prescription cannot be considered. Insufficient identification of the portion of land claimed in absolute ownership cannot ripen into ownership. Possession as a means of acquiring ownership, while it may be constructive, is not a mere fiction.37

Assuming, however, that the disputed land has been clearly identified, acquisitive prescription will still not lie in favor of the petitioners because they were not able to prove that they have been in possession of the property for the requisite number of

years. Prescription requires public, peaceful, uninterrupted and adverse possession of the property in the concept of an owner for ten years, in case the possession is in good faith and with just title.38

Aside from the testimony of Leonardo Vacalares that certain tenants of the petitioners cultivated the land for a total of seven years, the petitioners did not present any other evidence to show that they have been in actual possession of the property for at least ten years.

The petitioners’ argument that the payment of taxes on the property since May 31, 1948 constitutes proof of their possession of the subject land for thirty-five years is untenable. Tax declarations and receipts are not conclusive evidence of ownership. At most, they constitute mere prima facie proof of ownership of the property for which taxes have been paid. In the absence of actual, public and adverse possession, the declaration of the land for tax purposes does not prove ownership.39

IN LIGHT OF ALL THE FOREGOING, the petition is DENIED. The Decision of the Court of Appeals is AFFIRMED. No costs.

SO ORDERED.

Puno, J., Chairman, Austria-Martinez, Tinga, and Chico-Nazario, JJ., concur.

Page 20: Property Case

Republic of the PhilippinesSUPREME COURT

Manila

FIRST DIVISION

 

G.R. No. L-22006 July 28, 1975

BASILIO PEREZ and PETRA MONTALBO, petitioners, vs.NICOLAS MENDOZA, MARGARITA MACALALAD and the HONORABLE COURT OF APPEALS, respondents.

Pedro T. Panganiban for petitioners.

Julio D. Enriquez, Sr. for respondents.

 

MUNOZ PALMA, J.:

Civil Case 689 of the Court of First Instance of Batangas was an action to quiet title over a piece of land filed on March 20, 1959, by spouses Basilio Perez and Petra Montalbo with spouses Nicolas Mendoza and Margarita Macalalad as defendants. According to the complaint, the land in controversy is located in barrio Dagatan, municipality of Taysan, Batangas, with an area of approximately 4,765 sq. meters, declared for taxation purposes in the name of the "Heirs of Estanislao Montalbo", and is "bounded on the north by a school site, on the east by Calixto Flores, on the south by a creek, and on the west by a creek and the land of Gregorio Mendoza." On the basis of evidence adduced by the parties, the trial court then presided by Hon. Lorenzo Relova rendered judgment on February 19, 1962, dismissing the complaint and declaring the spouses Mendoza "to have a better right to the property in question." 1

Spouses Perez elevated the Relova decision to the Court of Appeals which, however, affirmed in toto the findings of the court a quo, and declared that "upon the evidence it has been shown by a great preponderance that the land in question belongs to the defendants." 2

The case is now before Us on a petition for certiorari filed by spouses Perez.

The findings of fact both of the trial court and the Court of Appeals may be briefly summarized as follows:

The litigated parcel of land was originally part of a bigger tract owned by Estanislao Montalbo. When Estanislao died in 1918, his properties passed on to his children Petra, Felisa, and Pedro all surnamed Montalbo, and because Pedro died single the two women remained as the only heirs. By mutual agreement Petra and Felisa divided between themselves the lands of their father and the parcel of which the litigated land was a part was assigned to Felisa. Sometime in 1922 Felisa exchanged the above-mentioned parcel with a land belonging to her aunt. Andrea Montalbo, a sister of her father. The reason for the exchange was that Andrea wanted to donate a piece of land to the municipality for use as a school site and the land of Felisa was what the municipality preferred as it was adjacent to other properties of the municipality. (Exh. 5 for defendants Mendoza) Upon her acquisition of Felisa's aforementioned land, Andrea donated to the municipality the northern portion thereof which constituted almost one-half of the entire parcel, and since then that portion was declared for taxation purposes by the municipality together with its adjoining properties (Exhs. 6, 6-A, 6-B).1äwphï1.ñët In 1927 the remainder of the lot was given by Andrea Montalbo to her daughter Margarita Macalalad on the occasion of her marriage to Nicolas Mendoza, and from the time of their marriage the couple

possessed the said property. That donation was confirmed subsequently in a public instrument dated August 15, 1951 (Exh. 2 for the Mendozas). Nicolas Mendoza sought to transfer the tax declaration of the property to his name and of his wife and for that purpose he submitted a deed of exchange of property dated January 14, 1922, allegedly executed by Felisa Montalbo and Andrea Montalbo in the presence of the municipal secretary Rafael Manahan (Exh. 5). When Basilio Perez came to know about the supposed deed of exchange, he had it investigated and upon discovering that the signature of Rafael Manahan appearing on the document was forged, he filed a criminal complaint before the Fiscal's office which led to an accusation for falsification of private document against Andrea Montalbo and Nicolas Mendoza. Only Nicolas Mendoza was arraigned and tried and was convicted by the Court of First Instance of Batangas, but on appeal he was acquitted by the Court of Appeals for insufficiency of evidence to show that he participated in affixing the signature of Rafael Manahan or that he was aware of the falsity of the document in question when he presented it to the tax assessor's office. 3 Notwithstanding the forged signature of Rafael Manahan on the document Exhibit 5, there is sufficient evidence to prove that an exchange of property did in fact occur in 1922 between Andrea and Felisa Montalbo, and that Felisa's land passed on to Andrea who in turn gave part of it to the municipality and part to her daughter, Margarita; hence, the decision in favor of the spouses Mendoza.

On the other hand, petitioners contend that the disputed property was inherited by Petra and Felisa Montalbo from their father Estanislao who died in 1918 and since that date the two sisters were in possession of said land. In 1934 a deed of partition of the various properties of Estanislao was executed between Petra and the heirs of Felisa, and the land in question was divided equally, between them; among those who signed as witnesses to that agreement was Andrea Montalbo(Exh. D for petitioners). In 1952 Felisa's husband, Jose Ortega, and children sold their one-half share to spouses Petra Montalbo and Basilio Perez, now petitioners, but the deed of sale was lost a year after. Sometime in 1946 petitioners leased the property to the Mendozas and when the lease expired in 1951 they demanded for the return of the land but the Mendozas refused and so petitioners had to file an ejectment suit before the justice of the peace court of Taysan which was still pending at the time of the trial of the civil case in 1960. (tsn. witness Basilio Perez, December 15, 1960, pp. 16-34)

For not giving credit to the foregoing evidence, petitioners now assail the adverse decision of respondent court on four assigned errors.

1. Petitioners contend that respondent court erred in considering the criminal case for falsification res adjudicata on the matter of ownership of the land in litigation when the "question of ownership was not actually and directly in issue in the criminal case and the latter was not the proper vehicle for the determination of the ownership of the land." (p. 9, petitioners brief) Petitioners refer to portions in the decision of respondent court, viz:

The land in question, together with that portion that was acquired by the municipality of Taysan, the identity of which is admitted by the parties, belonged to Felisa Montalbo, as held in the decision of the Court of Appeals, thus — "The said parcel of land previously belonged to Felisa Montalbo (married to Jose Ortega), who inherited it from her deceased father, the aforecited Estanislao Montalbo;", and the land in question was donated propter nuptias by Andrea Montalbo to Margarita Macalalad and Nicolas Mendoza, the defendants, (Margarita Macalalad is the daughter of Andrea Montalbo) on the occasion of their marriage on February 27, 1927, as found and held in the decision of the Court of Appeals, thus — "and this land was acquired by the donor (Andrea Montalbo) by

Page 21: Property Case

means of a barter with her own parcel of land planted with bamboos and mango trees"

Upon the basis of the findings of fact and conclusion arrived at in the decision of the Court of Appeals, it clearly appears that although the document of exchange of the lands was found to be falsified, nevertheless the Court found upon the facts as demonstrated by the evidence that the land in question "previously belonged to Felisa Montalbo (married to Jose Ortega), who inherited it from her deceased father, the aforesaid Estanislao Montalbo ..."; that said land was donated propter nuptias by Andrea Montalbo to the defendants on the occasion of their marriage on February 27, 1927; and that "this land was acquired by the donor by means of a barter with her own parcel of land planted with bamboos and mango trees". From the context of the decision the natural and logical inference is that factually the exchange of the lands had been consummated.... (pp. 6-7, CA decision at pp. 20-21, rollo; emphasis supplied to indicate disputed statements)

Undoubtedly, there is merit to the contention of petitioners that the pronouncements or findings of fact made by the Court of Appeals in the criminal case concerning the possession and ownership of the land now in litigation in the civil case, do not constitute the law on the matter and cannot be taken or adopted as a basis for deciding the question of ownership of said land in this civil case. Since there is no identity of parties in the two cases — the petitioners here not being parties in the criminal case — and the object or subject matter in the criminal prosecution is different, the latter being concerned with the guilt or innocence of accused Nicolas Mendoza for falsification of private document, it follows that the judgment in the criminal action cannot be used as evidence in the civil case where the issue is ownership of a piece of land. It is the rule that the plea of res judicata generally cannot be interposed except where the parties, facts, and questions are the same, 4 hence, the judgment in a criminal case cannot be pleaded as res judicata in a civil action. 5

But whatever error was committed by respondent court in this regard, the same is not sufficient to nullify the appealed decision.

Analyzing the decision of respondent court. We see that the latter made its own appraisal and evaluation of the evidence existing in the record relative to the possession and ownership of the land in question. Thus it said that the conclusions arrived at by the Court of Appeals in the criminal case to wit(1) that there was an exchange of lands consummated between Andrea and Felisa and (2) that the exchanged land was later donated by Andrea to her daughter Margarita in 1927, "can hardly be doubted if we take account of the undisputed fact that the defendants have been in possession of the land since 1927, and the plaintiffs (meaning spouses Perez) have not attempted to disturb defendants' possession of the land until 1952 when said plaintiffs filed an action of unlawful detainer against the defendants." (p. 7 of appealed decision at p. 21, SC rollo; emphasis supplied) Continuing, respondent court expounded:

Contrary to the allegation in the complaint — "That plaintiffs were in possession of the land prior and up to January, 1946, when the same was leased to the defendants ...", and the testimony of Basilio Perez to the same tenor, the evidence has conclusively shown that the defendants have been in continuous possession of the land since 1927 to the present time, and they have built a house on the land in 1928 where they have resided and lived to the present, as testified to by the defendant Mendoza, ....

The plaintiffs have contended, however, with the support of the testimony of Basilio Perez, that the possession of the defendants since 1946 was that of a mere lessee of the land. On this matter, the trial court said, "the records do not show any documentary evidence to support such contention. Nor is any document, say receipts of payment of rentals presented to bolster their theory. On the contrary their averment has been strongly denied by the defendants and the records show that it was only in 1952 that a civil action was instituted by the plaintiffs against the defendants in the Justice of the Peace Court of Taysan, Batangas, for detainer and damages", and said allegation of possession of the defendants as lessees of the land "is not supported by positive and convincing evidence". We find no reason to disagree with the foregoing findings of fact and conclusion of the trial court because the same is supported by the preponderance of evidence, and the plaintiffs have not pointed to us any fact of significance or influence which have been disregarded by the court, other than the testimony of Basilio Perez who testified about the supposed contract of lease. (pp. 21-22, 23, ibid.; emphasis supplied)

Digging further into the evidence of herein petitioners, respondent court found for itself that the agreement of partition dated May 27, 1934, Exhibit D, is not incontrovertible proof that in 1934 the litigated property belonged in common to Petra and the heirs of Felisa Montalbo both of whom may have been guided by the fact that the property was still declared for taxation purposes in the name of Estanislao Montalbo, and that the document of partition "did not overcome the evidence on record that Andrea Montalbo became the owner of the land, and that since 1927 the defendants have been in continuous possession of the land, openly, adversely and in the concept of owners thereby acquiring ownership of the land through acquisitive prescription." (p. 10 of CA decision at p. 24, SC rollo)

Independently therefore of the pronouncements of the Court of Appeals in the criminal case, respondent court examined the evidence in this civil case and made its own findings of fact on the basis of which it affirmed the decision of the trial court.

We could have stopped here and resolved this petition under well-entrenched precepts in Philippine jurisprudence that findings of fact of the Court of Appeals are as a rule conclusive and binding upon this Court; 6 nonetheless, to set our mind at rest that the conclusions of respondent court were not grounded on speculation, surmises or conjectures, 7 We went over the evidence before Us.

Certain salient facts strongly support the claim of respondents Mendoza over the property in dispute:

First, the northern boundary of the land in controversy is undisputably a school site which originally was part of a bigger tract belonging to Estanislao Montalbo. This is admitted by petitioner Basilio Perez who to a question propounded by his counsel, Atty. Panganiban, declared:

Mr. Panganiban: (Counsel of petitioners)

Q. According to these tax declarations which you said covers the land in question, the boundaries on the north, school site; on the east, land of Calixto Flores; on the south, estero; and on the west, estero and Gregoria Mendoza, why is it that there is a discrepancy?

A. Because from the whole parcel of land a portion was taken for the school site, and that

Page 22: Property Case

which remains now is the land in question, sir. (tsn December 15, 1960, pp. 22-23)

No explanation however was offered by Perez as to how that portion became a school site. On the other hand, there is evidence of respondent Mendoza that because Andrea Montalbo wanted to donate a piece of land to be used as a school site and the municipality preferred the location of the land inherited by Felisa from her father, the two women exchanged lands after which Andrea gave one-half of the property to the municipality while the remaining portion which is the land now in litigation was donated propter nuptias to her daughter Margarita way back in 1927. (tsn October 24, 1961, pp. 14-18) This donation of Andrea was not disproved by any evidence of petitioners. On the part of respondents Mendoza, their documentary evidence, Exhibits 6, 6-A and 6-B, show that the municipality of Taysan declared the donated property in its name as early as July, 1925, which supports respondents' claim that the exchange of properties between Andrea and Felisa Montalbo took place sometime in 1922.

Second, the provincial authorities authorities dealt with the Mendozas for the widening of the provincial road which traverses the land in question. Nicolas Mendoza testified that the land covered by the complaint actually consists of two lots which he described in his sketch, Exhibit 1, with letters "A" and "B" respectively, separated by a provincial road leading to the municipality of Lobo; that lot "A" which is the bigger parcel is the one donated to his wife, Margarita, by Andrea Montalbo on the occasion of their marriage in 1927 (Exh. 2); while lot "B" was bought from Donata Mendoza in 1951 as shown by the deed of sale, Exhibit 7; that sometime in 1937-38, the province widened the provincial road traversing the two lots, and he and his wife were approached by the provincial authorities more particularly, Engineer Ramirez, for them to give without compensation from lot "A" a stretch of land of one meter in width to widen said road, and they agreed. At that time Donata Mendoza still owned lot "B" and she was also asked to give part of her land for the road but she was paid for the value of the plants destroyed in the process.(tsn October 24, 1961, pp. 32-34) For his part, petitioner Perez admitted during the cross-examination conducted by the opposite counsel, Atty. Julio Enriquez, that the provincial authorities did not deal with him at all during the widening of that particular road. (tsn September 25, 1961, p. 34) This is of marked significance, because if it were true as claimed by petitioners that they were in possession of the property since the death of Estanislao Montalbo in 1918 or even after the deed of partition in 1934, they would have been the persons approached by the authorities for the widening of the road. The fact that the Mendozas were the ones who gave away part of the land for the widening of the Lobo road shows that they were in possession of the property and were living there at the time.

Third, respondents Mendoza have been in possession of the property since 1927 in concept of owners thereof. We have the testimony of respondent Nicolas Mendoza that after the land was donated to his wife in 1927 they built a house on it and lived there continuously, witness referring particularly to what he described as lot "A" in his sketch Exhibit 1. (tsn October 24, 1961, pp. 7, .30-31) Respondent's testimony was found both by the trial and appellate courts credible because (1) petitioner Basilio Perez himself admitted during cross-examination that even before the last world war the Mendozas had constructed a house on the land in litigation (tsn September 25, 1971, pp. 37-39; see Exh. E-3) which admission disproves the allegation in the complaint and Perez' testimony that it was only in 1946 when the Mendozas occupied the property as lessees; (2) the testimony of Nicolas Mendoza was corroborated by witness Adriano Gonzales, a retired justice of the peace of Taysan, Batangas, who declared that he knew the Mendozas since 1937 and he saw them living on the land in question and they have not changed residence at all since he had known them (tsn December 6, 1961, pp. 5-6); and (3) the respondents Mendoza were the ones who were living on the property and not the petitioners at

the time the provincial government in 1937 widened the Lobo road which crosses said land.

The court a quo and the respondent appellate court did not err when they upheld the claim of ownership of the Mendozas principally on the ground that the latter were in actual possession of the property since 1927 and were sought to be dispossessed by petitioners herein only in 1952 when an ejectment suit was filed against them.

Possession is an indicium of ownership of the thing possessed and to the possessor goes the presumption that he holds the thing under a claim of ownership. 8 Article 433 of the Civil Code provides that "(A)ctual possession under claim of ownership raises a disputable presumption of ownership. The true owner must resort to judicial process for the recovery of the property." In Chan vs. Court of Appeals, et al., L-27488, June 30, 1970, 33 SCRA 737, this Court upheld the finding of the Court of Appeals that the litigated property belonged to the private respondents therein based on their possession of the property, not only because such findings of fact of the appellate court are conclusive and binding on this Court but because the conclusion is in accordance with Articles 433 and 531 of the Civil Code. 9

As we have here conflicting claims of possession by the parties over the land in controversy and because the fact of possession cannot be recognized at the same time in two different personalities except in cases of co-possession, the present possessor is to be preferred pursuant to Article 538 of the Civil Code which We quote:

Possession as a fact cannot be recognized at the same time in two different personalities except in the cases of co-possession. Should a question arise regarding the fact of possession, the present possessor shall be preferred; if there are two possessors, the one longer in possession; if the dates of the possession are the same, the one who presents a title; and if all these conditions are equal, the thing shall be placed in judicial deposit pending determination of its possession or ownership through proper proceedings." 10

The pretension of petitioners that the possession of the Mendozas is that of a mere lessee was not believed by the trial judge and the appellate court not only because of the absence of any written or oral evidence on the matter other than the bare testimony of petitioner Basilio Perez, but also due to the circumstances present in the case which We indicated and enumerated at pages 7 to 9 of this decision. In fine, it is a fact that the Mendozas are presently in possession of the property and the presumption of ownership in their favor has not been successfully rebutted by evidence that they are mere lessees of the land in their possession as claimed by petitioners.

2. In their second assigned error, petitioners contend that respondent court should not have given weight to the evidence of respondent Mendoza because the latter's Exhibit 5 was proven to be a falsified document.

To recall, Exhibit 5 is the alleged deed of exchange or barter of lands between Andrea and Felisa Montalbo dated January 14, 1922. On this point, petitioners overlook the fact that Exhibit 5 was made the basis of a criminal accusation of falsification of private document solely on the allegation that the signature of Rafael Manahan, the person before whom the parties to the document allegedly appeared, was not his. There was no finding in that criminal case as per decision rendered therein that the barter or exchange of lands between Andrea and Felisa Montalbo did not in effect take place. On the contrary, what appears in said decision offered by petitioners as their Exhibit J are the following findings of the Court of Appeals, viz: that the land donated by Andrea Montalbo to her daughter Margarita Macalalad "was acquired by the donor by means of a barter with her own parcel of land planted with bamboos and mango trees"; that while it is true that because of this presentation of

Page 23: Property Case

the falsified document appellant (now respondent Nicolas Mendoza) was able to secure the declaration of the property donated in his name, no criminal liability should be imposed upon him in the absence of any evidence that he presented said exhibit with the knowledge that it was forged "especially if we take into consideration the fact that he and his wife were and are still in possession of the land donated since 1927"; that in fact, the color and appearance of the document in question show that it is not a new document but an old one thus confirming Mendoza's theory that it was executed in or about the year 1922 as appearing in the document or five years before his marriage. (pp. 1, 5, 6 of Exh. J, folder of exhibits) Thus, if the document Exhibit 5 was held to be forged, it was simply because the municipal secretary, Rafael Manahan, did not sign it and not for any other reason. What is material and relevant to the civil case is that both the trial court and respondent appellate court found for a fact that there was an exchange of lands between Andrea and Felisa Montalbo on the basis of evidence other than the disputed Exhibit 5. As to what the evidence is, has been discussed above.

Petitioners cite Gonzales vs. Mauricio, 53 Phil. 728 where this Court stated inter alia that the introduction of a forged instrument by a witness renders the testimony of the latter practically worthless. That statement however is not applicable to the situation before Us because in Gonzalez the particular document or receipt referred to was found to be entirely false as to its contents, handwriting, and signature, whereas here all that was found to be false is the signature of a witnessing official.

3. The last argument of petitioners is the object of the third assigned error. It is contended that the appellate court erred in not giving effect to the deed of partition, Exhibit D, notwithstanding the fact that the name of Andrea Montalbo appears in the document as one of the witnesses thereto.

Exhibit D appears to be a document dated May 27, 1934, wherein certain properties allegedly belonging to Estanislao Montalbo were divided between Petra Montalbo and Jose Ortega, husband of deceased Felisa Montalbo. Petitioner Basilio Perez declared that one of the parcels of land mentioned in the document is the land now in litigation which is particularly marked as Exhibit D-1. He also testified that Exhibit D was signed by him and his wife, Petra Montalbo, by Jose Ortega, husband of deceased Felisa Montalbo, and thumbmarked by the latter's children all in his presence. (tsn December 15,1960, pp. 19-24) Surprisingly, however, Basilio Perez did not at all mention during the course of his testimony that the old woman, Andrea Montalbo, signed the deed of partition as a witness. We have gone over the transcript of Basilio Perez' declaration on direct and cross-examination (tsn December 15, 1960, pp. 15-34; September 25, 1961, pp. 3-40) and at no instance did he ever state that Andrea Montalbo was present during the preparation of the document, that she read or knew the contents thereof which by the way consists of six handwritten pages, and that she signed her name on the document. It was incumbent upon petitioners to identify the signature of Andrea Montalbo on the document if her signature was truly there. As a matter of fact, examining the document Exhibit D We entertain doubts whether the name referred to by petitioners is "Andrea Montalbo", for, as written, it also can read "Maria Montalbo". At any rate, whatever is the import of said deed of partition, the same binds only the parties thereto but does not affect third persons such as Andrea Montalbo or the herein Mendozas in the absence of proof that they participated in one way or another in the preparation and execution of thedocument. As it is, Andrea Montalbo was a stranger to that deed of partition and any recital therein concerning the property under litigation cannot be used as evidence to prejudice her and her successors-in-interest or place her in estoppel as to her claims over the property. Res inter alios acta alteri nocere non debet. A transaction between two parties ought not to operate to the prejudice of a third person or stranger. 11

4. In the fourth assignment of error, petitioners claim that the appellate court should have rendered a decision in their favor. That both the trial court and respondent

appellate court have correctly evaluated the evidence, has been clearly demonstrated by Us.

IN VIEW OF ALL THE ABOVE CONSIDERATIONS, We find no reversible error in the decision under review and We AFFIRM the same with costs against petitioners.

So Ordered.

Castro (Chairman), Makasiar, Esguerra and Martin, JJ., concur.

Page 24: Property Case